Está en la página 1de 76

SERVICE TAX

INDEX

FINANCE ACT 1994


Extent, Commencement and Application Definition Classification of taxable services Charge of Service Tax Charge of service tax on services received from outside India Valuation of taxable services for charging service tax Payment of service tax Registration Furnishing of returns Scheme for submission of returns through service tax returns preparers Recovery of service tax not levied or paid or short-levied or short-paid or erroneously refunded Service tax collected from any person to be deposited with Central Government Interest on amount collected in excess Interest on delayed payment of service tax Penalty for failure to pay service tax Penalty for contravention of rules and provisions of Act for which no penalty is specified elsewhere. Penalty for suppressing, etc., of value of taxable services Penalty not to be imposed in certain cases Application of certain provisions of Central Excise Act, 1944 Appeals to the Commissioner of Central Excise (Appeals) Appeals to Appellate Tribunal Recovery of any amount due to Central Government Power to grant exemption from service tax Power to grant rebate (Duty drawback) in case of exports Power to make rules

SECTIONS 64 65 65A 66 66A 67 68 69 70 71 73 73A 73B 75 76 77 78 80 83 85 86 87 93 93A 94 Rule 2 4 5 6 7

SERVICE TAX RULES, 1994


Reverse Charge Registration Records Payments Return

POINT OF TAXATION RULES, 2011


Point of Taxation for Payment of Service Tax Point of Taxation for change in rate of service tax Point of Taxation for Service Tax on New Services Point of Taxation for continuous supply of Services Payment of Service Tax on Receipt Basis in certain cases

Rule 3 4 5 6 7

Q1: Explain Selective Approach/ Comprehensive Approach to Service Tax. Q2: Explain meaning of Taxable Service. Q3 (Imp.): Write a note on Constitutional validity of Service Tax. Q4 (V. Imp.): Write a note on sources of Service Tax Law. Q5: Write a note on Administration of Service Tax. Q6: Write a note on Service Tax Procedures. Q7 (Imp.): Write a note on role of a Chartered Accountant in service tax. Q8 (V. Imp.): Write a note on Extent, Commencement and Application of Service Tax. Q9: Is service tax destination based consumption tax? Q10: Write a note on charge of Service Tax. Q11 (V. Imp.): What is the exemption limit for the Small Service Provider. Q12 (V. Imp.): Write a note on General Exemptions under Service Tax. Q13 (V. Imp.): Write a note on Registration under Service Tax. Q14 (V. Imp.): Write a note on Centralized Registration under Service Tax. Q15: Write a note on Amendment of Registration Certificate. Q16: Write a note on Cancellation of Registration Certificate. Q17: Write a note on issue of Invoice/Bill by Service Provider. Q18: Explain Determination of Rate for Service Tax. Q19 (V. Imp): Write a note on payment of Service Tax. Q20 (V. Imp.): Write a note on Point of Taxation for Payment of Service Tax. Q21: Write a note on interest on delayed payment of Service Tax. Q22: Write a note on penalty for failure to pay Service Tax. Q23: Write a note on penalty for contravention of rules and provisions of Act for which no penalty is specified elsewhere. Q24: Write a note on penalty for suppressing (to conceal) value of taxable services. Q25 (Imp.): Write a note on Advance Payment of Service Tax. Q26 (Imp.): Write a note on Refund of Service Tax/ Issue of Credit Note. Q27 (Imp.): Write a note on Provisional Payment of Service Tax. Q28 (V. Imp.): Write a note on Self Adjustment of Service Tax. Q29 (Imp.): Explain payment of service tax in case of Associated Enterprises. Q30: Write a note on adjustment for Municipal Tax paid for House Property. Q31 (Imp.): Write a note on service tax collected wrongly or in excess from any person. Q32 (Imp.): Write a note on point of taxation for determining the rate of Service Tax. Q33: Write a note on liability to service tax in case of New Services. Q34: What are the provisions of service tax for Continuous Supply of Service.

Q35: Write a note on determination of Point of Taxation in case of Copyrights, etc. (For Self Reading) Q36 (V. Imp.): Write a note on Filing of Service Tax Return. Q37: Write a note on Revision of Service Tax Return. Q38 (V. Imp.): Write a note on Delay Furnishing of Service Tax Return. Q39: Write a note on contents of the Service Tax Return. Q40: Write a note on documents to be submitted along with Service Tax Return. Q41 (V. Imp.): Write a note on E-Filing of Service Tax Return. Q42: Write a note on Service Tax Return Preparer Scheme. Q43: Write a note on Records under Service Tax. Q44 (V. Imp.): Write a note on option to pay service tax in case of Air Travel Agent. Q45 (V. Imp.): Write a note on option to pay service tax by any insurer carrying on Life Insurance Business. Q46 (Imp.): Write a note on option to pay service tax in case of Money Changer. Q47: Write a note on option to pay service tax in case of Distribution of Lottery ticket. Q48 (V. Imp.): Write a note on Reverse Charge Mechanism. Q49 (V. Imp.): Write a note on Insurance Auxiliary (help) Service. Q50 (V. Imp.): Write a note on Import of Services. Q51 (V. Imp.): Write a note on Goods Transport Agency. Q52: Write a note on Business Auxiliary Services of Distribution of Mutual Fund. Q53 (V. Imp.): Write a note on Sponsorship Services. Q54 (V. Imp): Write a note on tax credit in case of Service Provider. Q55: Explain provisions of Rule 6 of Cenvat Credit Rules, 2004. Q56: Whether any section of Central Excise Act, 1944 is applicable to Service tax. (For Self Reading Only) Q57: Write a note on Appeals to the Commissioner of Central Excise (Appeals). (For Self Reading Only) Q58: Write a note on Appeals to Appellate Tribunal (CESTAT). (For Self Reading Only) Q59 (V. Imp.): Write a note on valuation of taxable services for charging Service Tax. Q60: Explain the concept of Pure Agent. Q61 (V. Imp.): Write a note on Export of Services. Q62 (V. Imp.): Explain meaning of Input Service / Output Service. Q63 (Imp.): Write a note on Input Service Distributor. Q64 (Imp.): Write a note on liability of service tax in case of supply of goods as well as services. Q65 (Imp.): Explain concept of Partial Abatement. Q66: Write a note on services rendered by Sub-Contractor. Q67: Explain Recovery of Service Tax. (Self reading) Q68: Explain provisions of Tax Deduction at Source (TDS) with regard to Service Tax. Q69 (V. Imp.): Explain provisions relating to Consulting Engineers Services. Q70 (V. Imp.): Explain provisions relating to Mandap Keepers Service. Q71 (V. Imp.): Explain provisions relating to Practising Chartered Accountant. Q72 (V. Imp.): Explain provisions relating to Scientific and Technical Consultancy Service. Q73 (V. Imp.): Explain provisions relating to Commercial Training or Coaching. Q74 (V. Imp.): Explain provisions relating to Technical Testing and Analysis Service. Q75 (V. Imp.): Explain provisions relating to Business Exhibition Service. Q76 (V. Imp.): Explain provisions relating to Information Technology Software Service.

Service Tax in India The genesis of service tax is the result of recommendations made in 1990s by the Tax Reforms Committee headed by Professor Dr. Raja J. Chelliah. Based on the above recommendations, Dr. Manmohan Singh, the Union Finance Minister, introduced the concept of service tax in 1994-95 w.e.f. 01.07.1994. Initially service tax was imposed only on three services i.e. telephones, non-life insurance and stock brokers but at present there are 119 services which are chargeable to service tax and are called Taxable Services and details are given in section 65(105). Question 1: Explain Selective Approach/ Comprehensive Approach to Service Tax. Answer: Each country has a taxation system on services adopting either a Comprehensive Approach or a Selective Approach. In Comprehensive Approach all services are made taxable and a list is given in case some services are to be exempted. In Selective Approach, selective services are subjected to service tax. While most of the developed countries tax all the services with very few and limited exemptions, most of the developing countries have opted for taxation of select services only. India has adopted a selective approach to taxation of services and the services which are chargeable to service tax as given in section 65(105) of Finance Act, 1994. Question 2: Explain meaning of Taxable Service. Answer: The services which are chargeable to service tax are called Taxable Service and the services which are not chargeable to service tax are called Exempted Services and at present there are 119 services which are Taxable Services and are given under section 65(105) of Finance Act, 1994 and are as given below: (Students need not learn the list given below rather it is only for reading) S. Taxable Service Section Effective No from . 1 Stock Broking 65(105)(a) 01.07.1994 2. General Insurance 65(105)(d) 01.07.1994 3. Advertising Agency Services 65(105)(e) 01.11.1996 4. Courier 65(105)(f) 01.11.1996

5. 6. 7. 8. 9. 10. 11. 12. 13. 14. 15. 16. 17. 18. 19. 20. 21. 22. 23. 24. 25. 26. 27. 28. 29. 30. 31. 32. 33. 34. 35. 36. 37. 38. 39. 40. 41. 42. 43. 44. 45. 46. 47. 48.

Consulting Engineer Custom House Agent Steamer Agent Clearing & Forwarding Agent Manpower Recruitment or Supply Agency Air Travel Agent Mandap Keeper Tour Operator Rent-a-Cab Operator Architect Interior Decorator Management or Business Consultant Practicing Chartered Accountants Services Practicing Cost Accountants Services Practicing Company Secretary Services Real Estate Agent Security Agency Credit Rating Agency Market research agency Underwriter Scientific and Technical Consultancy Photography Convention On-line information and data base access or retrieval(computer network) Video Production Agency Sound Recording Broadcasting Insurance Auxiliary (General Insurance) Banking and Financial services Port Authorised service station (Motor Car / Two- wheelers /LMV) Beauty Treatment Cargo Handling Cable Dry Cleaning Event Management Fashion Designing Health and Fitness Life Insurance Insurance Auxiliary (life insurance) Rail Travel Agent Storage and Warehousing Business Auxiliary Services Commercial Training or Coaching

65(105)(g) 65(105)(h) 65(105)(i) 65(105)(j) 65(105)(k) 65(105)(l) 65(105)(m) 65(105)(n) 65(105)(o) 65(105)(p) 65(105)(q) 65(105)(r) 65(105)(s) 65(105)(t) 65(105)(u) 65(105)(v) 65(105)(w) 65(105)(x) 65(105)(y) 65(105)(z) 65(105)(za) 65(105)(zb) 65(105)(zc) 65(105)(zh) 65(105)(zi) 65(105)(zj) 65(105)(zk) 65(105)(zl) 65(105)(zm) 65(105)(zn) 65(105)(zo) 65(105)(zq) 65(105)(zr) 65(105)(zs) 65(105)(zt) 65(105)(zu) 65(105)(zv) 65(105)(zw) 65(105)(zx) 65(105)(zy) 65(105)(zz) 65(105)(zza) 65(105)(zzb) 65(105)(zzc)

07.07.1997 15.06.1997 15.06.1997 16.07.1997 07.07.1997 01.07.1997 01.07.1997 01.09.1997 16.07.1997 16.10.1998 16.10.1998 16.10.1998 16.10.1998 16.10.1998 16.10.1998 16.10.1998 16.10.1998 16.10.1998 16.10.1998 16.10.1998 16.07.2001 16.07.2001 16.07.2001 16.07.2001 16.07.2001 16.07.2001 16.07.2001 16.07.2001 16.07.2001 16.07.2001 16.07.2001 16.08.2002 16.08.2002 16.08.2002 16.08.2002 16.08.2002 16.08.2002 16.08.2002 16.08.2002 16.08.2002 16.08.2002 16.08.2002 01.07.2003 01.07.2003

49. 50. 51. 52. 53. 54. 55. 56. 57. 58. 59. 60. 61. 62. 63. 64. 65. 66. 67. 68. 69. 70. 71. 72. 73. 74. 75. 76. 77. 78. 79. 80. 81. 82. 83. 84. 85. 86. 87. 88. 89. 90. 91.

Erection, Commissioning or Installation Franchise Internet Caf Management, Maintenance or Repair Technical Testing and Analysis Technical Inspection and Certification Foreign Exchange Broking Other Ports Airport Services Transport of Goods by Air Business Exhibition Transport of Goods by Road Commercial or Industrial Construction Intellectual Property Opinion Poll Outdoor Caterer Programme Production (of TV or Radio Programmes) Survey and Exploration of Mineral Pandal or Shamiana Travel Agent (other than Air Travel Agent and Rail Travel Agent) Forward Contract Transport of Goods (other than water) through Pipeline or Conduit Site Formation and Clearance etc. Dredging (to remove mud, stones, etc.) Survey and Map-Making Cleaning Activity Club or Associations Services Packaging Activity Mailing list Compilation and Mailing Construction of (Residential) Complex Registrar to An Issue Share Transfer Agent ATM Operations, Maintenance or Management Recovery Agent Sale of Advertising Space or Time Sponsorship Service Air Transport of Passengers Embarking (to get onto a ship) for International Travel Transport of Goods by Rail Business Support Services Auctioneers Service Public relations Management Service Ship Management Services Internet Telecommunication

65(105)(zzd) 65(105)(zze) 65(105)(zzf) 65(105)(zzg) 65(105)(zzh) 65(105)(zzi) 65(105)(zzk) 65(105)(zzl) 65(105)(zzm) 65(105)(zzn) 65(105)(zzo) 65(105)(zzp) 65(105)(zzq) 65(105)(zzr) 65(105)(zzs) 65(105)(zzt) 65(105)(zzu) 65(105)(zzv) 65(105)(zzw) 65(105)(zzx) 65(105)(zzy) 65(105)(zzz) 65(105)(zzza) 65(105)(zzzb) 65(105)(zzzc) 65(105)(zzzd) 65(105)(zzze) 65(105)(zzzf) 65(105)(zzzg) 65(105)(zzzh) 65(105)(zzzi) 65(105)(zzzj) 65(105)(zzzk) 65(105)(zzzl) 65(105)(zzzm) 65(105)(zzzn) 65(105)(zzzo) 65(105)(zzzp) 65(105)(zzzq) 65(105)(zzzr) 65(105)(zzzs) 65(105)(zzzt) 65(105)(zzzu)

01.07.2003 01.07.2003 01.07.2003 01.07.2003 01.07.2003 01.07.2003 01.07.2003 16.07.2001 10.09.2004 10.09.2004 10.09.2004 01.01.2005 10.09.2004 10.09.2004 10.09.2004 10.09.2004 10.09.2004 10.09.2004 10.09.2004 10.09.2004 10.09.2004 16.06.2005 16.06.2005 16.06.2005 16.06.2005 16.06.2005 16.06.2005 16.06.2005 16.06.2005 16.06.2005 01.05.2006 01.05.2006 01.05.2006 01.05.2006 01.05.2006 01.05.2006 01.05.2006 01.05.2006 01.05.2006 01.05.2006 01.05.2006 01.05.2006 01.05.2006

92. 93. 94. 95. 96. 97. 98. 99. 10 0. 10 1. 10 2. 10 3. 10 4. 10 5. 10 6. 10 7. 10 8. 10 9. 11 0. 11 1. 11 2. 11 3. 11 4. 11 5. 11 6. 11 7.

Transport of Person by Cruise Ships (to travel in a ship or boat visiting different places, especially as a holiday/vacation) Credit card, debit card, charge or other payment cards related services Telecommunication Services Mining of Mineral, Oil or Gas Services Renting of Immovable Property Works Contract Services Development and Supply of Content Services Asset Management Including Portfolio Management Design Services Information Technology Software Unit Linked Insurance Plan (ULIP) Management Stock Exchange Commodity Exchange Processing and Clearing House Supply of Tangible Goods for Use Cosmetic and Plastic Surgery Transport of Goods Through Waterways Legal Consultancy Promotion, Marketing, Organizing Games Clinical Establishment Service Medical Record Agency Promotion or Marketing of Brand Granting Right or Commercial Use of an Event Electricity Exchange Services Copyright Construction Services Preferential Location and Development

65(105)(zzzv) 65(105)(zzzw) 65(105)(zzzx) 65(105)(zzzy) 65(105)(zzzz) 65(105)(zzzza) 65(105)(zzzzb) 65(105)(zzzzc) 65(105)(zzzzd) 65(105)(zzzze) 65(105)(zzzzf) 65(105)(zzzzg) 65(105)(zzzzh) 65(105)(zzzzi) 65(105)(zzzzj) 65(105)(zzzzk) 65(105)(zzzzl)

01.05.2006 01.05.2006 01.06.2007 01.06.2007 01.06.2007 01.06.2007 01.06.2007 01.06.2007 01.06.2007 16.05.2008 16.05.2008 16.05.2008 16.05.2008 16.05.2008 16.05.2008 01.09.2009 01.09.2009

65(105)(zzzzm) 01.09.2009 65(105)(zzzzn) 65(105)(zzzzo) 65(105)(zzzzp) 65(105)(zzzzq) 65(105)(zzzzr) 65(105)(zzzzs) 65(105)(zzzzt) 65(105)(zzzzu) 01.07.2010 01.07.2010 01.07.2010 01.07.2010 01.07.2010 01.07.2010 01.07.2010 01.07.2010

11 8. 11 9.

Restaurant Service Short-Term Accommodation /Hotel Accommodation

65(105) (zzzzv) 01.05.2011 65(105)(zzzzw) 01.05.2011

Question 3 (Imp.): Write a note on Constitutional validity of Service Tax. Answer: The Government derives its power to levy taxes from Constitution of India. As per article 265, no tax shall be levied or collected except by authority of law. As per Article 246, the Constitution in its Schedule VII has mentioned the matters on which Central Government and State Government can make laws. Such matters are divided into three categories. List I: Union List (It contains the matters in respect of which only the Central Government can make law). (Total Entries 97) List II: State List (It contains the matters in respect of which only the State Government can make (Total Entries 66) List III: Concurrent List (It contains the matters in respect of which both the Central Government and State Government have power of legistration). (Total Entries 46)

law). the

Initially there was no specific entry in the Union List for levying service tax. Service tax was levied by the Central Government by drawing power from Entry 97 of the Union List. Entry 97 is a residuary entry in List-I, which has been reproduced below: 97 Any other matter not enumerated in List II or List III including any tax not mentioned in either of those Lists. The residuary entry provides wide power to the Central Government in respect of taxation of the subjects not mentioned in the Lists given by the constitution. Entry 92C was introduced in the VII schedule in the Union List with effect from 07.01.2004 vide article 268A. Entry 92C reads as under: 92C Taxes on services But till date this entry (92C) is kept dormant (ineffective). Question 4 (V. Imp.): Write a note on sources of Service Tax Law. Answer: Sources of Service Tax Law are as given below: 1. Finance Act, 1994 2. Rules 3. Notifications 4. Circulars 5. Trade Notices 1. Finance Act, 1994 The provisions relating to service tax are given in chapter V of Finance Act, 1994. Later, Finance Act, 2003 inserted chapter VA in Finance Act, 1994. It contains section 64 to 96-I. In the subsequent years additional provisions were introduced. So far there is no Service Tax Act. In the year 2004, the provisions relating to charging of education cess on the service tax were made applicable through Chapter VI of the Finance (No.2) Act, 2004. Thereafter secondary and higher education cess was levied through Chapter VI of Finance Act, 2007. 2. Rules on Service Tax Section 94/96-I of Finance Act, 1994 grants power to the Central Government to make rules for service tax. Central

Government has made rules as given below: (i) Service Tax Rules, 1994 (ii) Point of Taxation Rules, 2011 (iii) Service Tax (Determination of Value) Rules, 2006 (iv) Taxation of Services (Provided from Outside India and Received in India) Rules, 2006. (v) Export of Service Rules, 2005 (vi) Service Tax Return Preparer Scheme, 2009 (vii) Works contract (Composition Scheme for Payment of Service Tax) Rules, 2007 (viii) Service Tax (Registration of Special Category of Persons) Rules, 2005 (ix) CENVAT credit Rules, 2004 3. Notifications on Service Tax Section 93, 94, 96-I of Finance Act, 1994 empowers Central Government to issue notifications to exempt any service from service tax. 4. Circulars and office letters on Service Tax The Central Board of Excise and Custom issues circular and instruction to explain scope of taxable services and the scheme of service tax administration. The circulars bring out the real intention of the legislature. 5. Trade Notices on Service Tax Trade notice are issued by the commissioners for effective administration of service tax law. The trade notice are circulated among the field officers and also copies are given to Trade Associations. Question 5: Write a note on Administration of Service Tax. Answer: Ministry of Finance has two main Departments 1. Department of Revenue 2. Department of Expenditure All the taxes are covered under the Department of Revenue and the Department of Revenue has two Board of Taxes: 1. Central Board of Direct Taxes 2. Central Board of Excise and Customs Service Tax is administered by CBEC and the administration of service tax is as given below in the descending (decreasing) order: 1. Central Board of Excise and Customs 2. Chief Commissioners of Central Excise 3. Commissioners of Central Excise [Director General of Service Tax (Co-ordinator between 1 & 3)] The work relating to service tax is also under the control of CBEC, however in 1997 a new post of Director General (Service Tax) was formed. The functions and power of Direct General (Service Tax) are as follows: (1) To monitor the collection and assessment of service tax.

(2) To study the staff requirement at field level. (3) To increase revenue collection or to streamline procedures. (4) To undertake study of law and procedures in relation to service tax with a view to simplify the service tax collection and assessment and make suggestions thereon. (5) To form a data base regarding the collection of service tax. (6) To inspect the service tax cells in the Commissionerate. (7) To undertake any other functions as assigned by the Board from time to time. Question 6: Write a note on Service Tax Procedures. Answer: Service tax procedures includes (i) Registration (ii) Maintenance of books and records (iii) Payment of service tax (iv) Availment and utilization of CENVAT credit (v) Filing of service tax returns (vi) Assessment and recovery of service tax (vii) Interest and penalties (viii) Rectification of mistakes (ix) Revision of assessment order (x) Appeals (xi) Search and seizure (xii) Advance rulings etc. Question 7 (Imp.): Write a note on role of a Chartered Accountant in service tax. Answer: A Chartered Accountant is well equipped to position himself in the new role as an advisor and facilitator for due compliance of service tax law. The nature of services are: 1. Advisory services: A Chartered Accountant is considered to be qualified and competent to comprehend (understand) and interpret the service law to give required advice and guidance. 2. Procedural compliance: The service tax law requires Registration, Payment of tax, Filing of returns etc. A Chartered Accountant is well suited to assist the assessee in all the above functions and ensure compliance. 3. Personal representation: A Chartered Accountant is allowed to appear before the assessment authority, Commissioner (Appeals) and Custom, Excise and Service Tax Appellate Tribunal. With his expertise a Chartered Accountant can represent his clients. 4. Certification and audit: Service tax Returns and Financial Statements can be certified by the Chartered Accountant.

Question 8 (V. Imp.): Write a note on Extent, Commencement and Application of Service Tax. Answer: Extent, Commencement and Application Section 64 The Finance Act, 1994 came into force from 01.07.1994. The Act extends to the whole of India except the state of Jammu and Kashmir. Thus, services provided in the state of Jammu and Kashmir are not liable to service tax. As per Article 370 of the Constitution, any Act of Parliament applies to Jammu & Kashmir only with concurrence (approval) of State Government. Since, no such concurrence has been obtained in respect of Finance Act, 1994, service tax provisions are not applicable in Jammu and Kashmir. Service tax will not be payable if service is provided in Jammu & Kashmir, however, if a person from Jammu & Kashmir provides the service outside Jammu & Kashmir, the service will be liable to service tax. Merely because the office of the service provider is situated in Jammu & Kashmir, it does not mean that service is provided in Jammu & Kashmir. Service provided within the Territorial Waters will be liable to service tax, as levy of service tax extends to whole of India except Jammu and Kashmir and India includes territorial waters. Indian territorial waters extend upto 12 nautical miles from the Indian land mass. (1 nautical mile = 1.852 km) The provisions are applicable even in the Exclusive Economic Zone of India i.e. the provisions are applicable upto 200 nautical miles from the baseline (Notification no. 1/2002 dated 01.03.2002) As per Notification No. 21/2009 ST, dated 07.07.2009, Service provided to or from Installations, Structures and Vessels in entire Continental Self and Exclusive Economic Zone of India have been brought within the ambit of provisions relating to Service Tax. Question 9: Is service tax destination based consumption tax? Answer: True, service tax is a destination based consumption tax. It means that tax shall be payable only if service tax is imposed at the place where the service is used. Service tax is applicable all over India except the state of J & K. Therefore if services are used in J & K or outside India then it shall not be taxable. Example Mr. A of Delhi provided service to Mr. B of J & K. the service was provided and used in J & K. No service tax shall be payable in such cases. Example Salman and Associates is a firm of Chartered Accountant situated in the state of Jammu and Kashmir and provides accounting services to A Ltd. of Delhi. In this case, Salman and Associates is liable to pay service tax. As per section 64 The provisions of service tax apply to whole of India, except Jammu and Kashmir As service tax is a destination based consumption tax.

Therefore services consumed in J & K are not liable to service tax. But in the given case the services are provided and consumed in Delhi (i.e. outside J & K) Hence Salman and Associates will be liable to charge tax from A Ltd. Example Bharti Ltd. is a company which provides telecommunication services under the brand name of Airtel. The Company has its registered office situated in Delhi. Mr. A is a resident of Jammu and is a subscriber of mobile connection of Airtel. In this case, Bharti Ltd. cannot charge service tax from Mr. A. Service tax is a destination based consumption tax. It is immaterial that where the office of the service provider is located. If service is received and consumed in J & K then it shall not be taxable. Therefore, Mr. A subscribing to the services of Airtel in Jammu and Kashmir will not be liable to pay tax. Question 10: Write a note on charge of Service Tax. Answer: Charge of Service Tax Section 66 Section 66 is the charging section which deals with levy and collection of service tax. It prescribes the rate applicable for levying of service tax. The prescribed manner for collection and payment of service tax is provided in the Service Tax Rules, 1994. The rate of service tax at present is 10% and education cess and secondary and higher education cess is payable @ 3% on service tax. Service tax is payable on the value of taxable services and valuation is given in section 67 of the Finance Act, 1994.E.g. If a service provider has issued a bill of `10,00,000, service tax shall be `10,00,000 x 10.3% = `1,03,000 Alternative rate of service tax in case of certain services Although service tax is levied at the basic rate of 10% but in case of certain services, an alternative rate is also provided and is asunder: (1) In case of Life Insurance Services alternate mode of discharge of service tax liability has been provided and the rate of service tax in this case is 1.5% plus EC of total premium charged from the policy holders. (2) In case of Air Travel Agents Services, the Air Travel Agent, shall have the option of pay service tax of an amount calculated as under: (a) In case of domestic bookings 0.6% plus EC of the basic fare. (b) In case of international bookings 1.2% plus EC of the basic fare (3) In case of purchase and sale of foreign currency provided by foreign exchange broker, etc. provided consideration for the service provided is not shown separately in the invoice (a) 0.1% of the gross amount of currency exchanged upto ` 100,000, but minimum ` 25; (b) on next 9,00,000 @ 0.05% (c) on balance @ 0.01% but maximum `5,000. Question 11 (V. Imp.): What is the exemption limit for the Small Service Provider. Answer: As per Notification No.6/2005, dated 01.03.2005, exemption is allowed upto `10,00,000 provided in the preceding year,

value of taxable services rendered is not exceeding `10,00,000. While computing amount of `10,00,000, value of services which are exempt from service tax shall not be considered. If any person is rendering services from more than one premises, value of services rendered from all the premises shall be taken into consideration. If any person is rendering more than one services, value of all the services shall be taken into consideration. If any abatement is available, value before permitting abatement shall be taken into consideration. Example If a Mandap Keeper has started rendering services in the Financial Year 2011-12 and he is providing catering services also and the amount to be received for the services rendered by him is `12,00,000, in this case, abatement shall be allowed for 40% and remaining 60% is taxable and in order to check the limit of 10,00,000, gross amount before abatement shall be taken into consideration and in this case service tax shall be payable with regard to `2,00,000 but after permitting abatement @ 40% and Mandap Keeper should charge service tax in the manner given below: Gross amount 12,00,000 Less: Exemption 10,00,000 Balance amount 2,00,000 60% of balance amount 1,20,000 Amount of service tax @ 10.3% 12,360 No CENVAT Credit is allowed till the service provider is availing exemption under Notification No. 6/2005. Example A Chartered Accountant has started his practice wef 01.04.2011 and amount received upto 31.12.2011 is `10,00,000 and he has taken input services of `3,00,000 and paid service tax of `30,900, in this case, cenvat credit for input services is not allowed and also he cannot charge service tax on `10,00,000. If he has rejected general exemption of `10,00,000, in that case he should charge service tax of 10,00,000 x 10.3% = `1,03,000 and also cenvat credit is allowed for input services amounting to `30,900 and net tax payable shall be `72,100 The service provider has the option not to avail exemption and such option can not be withdrawn during the relevant year, however, it can be withdrawn from the beginning of next year. If any person is registered as per section 68(2), rule 2(1)(d), no such exemption shall be allowed and tax shall be payable right from the beginning i.e. in case of Reverse Charge mechanism, general exemption of `10,00,000 is not allowed. Question 12 (V. Imp.): Write a note on General Exemptions under Service Tax. Answer: 1. Exemption to all services provided to United Nations or International Organisation as per Notification No 16/2002 dated 02.08.2002 Central Government has exempted all the taxable services specified in section 65(105) of the Act provided by any person to the United Nations or an International Organisation, from whole of the service tax leviable thereon under section 66 of the Act. International Organisation means an International Organisation declared by the Central Government.

2. Exemption to services provided to a Developer or Units of Special Economic Zone as per Notification No. 17/2011 dated 01.03.2011 Taxable service provided to a developer of special economic zone or a unit (including a unit under construction) of special economic zone by any service provider, for consumption of the services within such special economic zone, are exempt from the whole of service tax. If the services are consumed outside SEZ, the service provider should charge service tax however, the recipient can claim refund. 3. Exemption to service provided by or to the Reserve Bank of India as per Notification No. 22/2006 ST dated 31.05.2006 Following taxable services are exempted from the whole of the service tax leviable thereon: (i) taxable services provided to any person, by the Reserve Bank of India; (ii) taxable services provided by any person, to the Reserve Bank of India when the service tax for such is liable to be paid by the Reserve Bank of India i.e. if reverse charge is not applicable, service tax shall be charged by the service provider from Reserve Bank and shall be paid to the Government. (iii) taxable services received in India from outside India by the Reserve Bank of India. 4. Service tax not leviable on fee collected by Public Authorities while performing Statutory Functions/ Duties under the provisions of law as per Circular No. 89/7/2006 ST dated 18.12.2006 / 96/7/2007 ST dated 23.08.2007 Service tax shall not be leviable on fee collected by public authorities while performing statutory functions/duties. A number of sovereign/public authorities (i.e. an agency constituted/set up by government) perform certain functions/duties, which are statutory in nature. For example the Regional Transport Officer (RTO) issues fitness certificate to the vehicles; or Explosive Department inspects and issues certificate for petroleum storage tank, LPG/CNG tank in terms of provisions of the relevant laws. Such activity is purely in public interest and it is undertaken as mandatory and statutory function. Therefore, no service tax is leviable on such activities. 5. Exemption to services provided to Foreign Diplomatic Mission or Consular Post in India as per Notification No. 33/2007 dated 23.05.2007 6. Exemption to services provided for personal use or for use of Family Members of Diplomatic Agents or Career Consular Officers posted in Foreign Diplomatic Mission / Consular Post in India as per Notification No. 34/2007 dated 23.05.2007 7. Exempts all taxable services, provided or to be provided, by a Technology Business Incubator (TBI) or a Science and Technology Entrepreneurship Park (STEP) Recognized by the National Science and Technology Entrepreneurship Development Board (NSTEDB) of the Department of Science and Technology, Government of India, from the service tax.

As per Notification No. 9/2007, the Central Government, exempts all taxable services, provided or to be provided, by a Technology Business Incubator (TBI) or a Science and Technology Entrepreneurship Park (STEP) recognized by the National Science and Technology Entrepreneurship Development Board (NSTEDB) of the Department of Science and Technology, Government of India, from the whole of the service tax leviable thereon under section 66 of the said Finance Act. The STEP or the TBI, who intends to avail the exemption, shall furnish the requisite information in prescribed format containing the details of the incubator along with the information in prescribed format received from each incubatee to the concerned Assistant Commissioner or the Deputy Commissioner of Central Excise, as the case may be, before availing the exemption; and 8. Exempts taxable services provided by an entrepreneur located within the premises of a Technology Business Incubator (TBI) or Science and Technology Entrepreneurship Park (Step) Recognized by the National Science and Technology Entrepreneurship Development Board (NSTEDB) of the Department of Science and Technology Government of India from the service tax leviable. As per Notification No. 10/2007, Central Government, exempts taxable services provided by an entrepreneur located within the premises of a Technology Business Incubator (TBI) or Science and Technology Entrepreneurship Park (STEP) recognized by the National Science and Technology Entrepreneurship Development Board (NSTEDB) of the Department of Science and Technology Government of India from the whole of the service tax leviable thereon under section 66 of the said Finance Act subject to the following conditions, namely:(i) the entrepreneur enters into an agreement with the TBI or the STEP as an incubatee, to enable himself to develop and produce hi-tech and innovative products; and (ii) Exemption shall apply for a period of three years from the date on which such entrepreneur enters into an agreement with the TBI or the STEP. (iii) The exemption shall not be available immediately after the total business turnover of the entrepreneur exceeds fifty lakh rupees during a given financial year. Question 13 (V. Imp.): Write a note on Registration under Service Tax. Answer: Registration Section 69/Rule 4(1) of STR, 1994 A service provider can charge service tax only if he is registered with the Service Tax Department. As per Notification No. 26/2005 dated 07.06.2005, every service provider in whose case, gross receipt has exceeded `9,00,000 shall apply for registration to the Service Tax Department in Form No ST-1 within 30 days from the date of crossing the limit of `9,00,000, however, he will charge service tax after crossing the limit of `10,00,000. He will also submit a copy of Permanent Account Number and proof of residence. Department shall grant him a registration certificate in Form No ST-2 within 7 days from the date of receipt of application otherwise the service provider shall be deemed to be registered. The service provider shall be given a Registration Number by the Department which will be called STP code i.e. Service

Tax Payer Code and it will be 15 digit PAN based number and first 10 digit shall be that of PAN and remaining 5 digit shall be allotted by Service Tax Department e.g. AAEPC1298D ST-001 The last three digit shall indicate total number of registration for the same permanent Account Number. If a person is providing more than one taxable service, he may give only one application. He should mention in the application all the taxable services provided by him. A person may apply for voluntary registration at any time and also a person may forgo (reject) the general exemption of `10,00,000. As per section 77, any person who has failed to take registration shall pay penalty of `200 per day or `10,000 whichever is higher. Question 14 (V. Imp.): Write a note on Centralized Registration under Service Tax. Answer: Centralised Registration Rule 4(2) of STR, 1994 If a service provider is providing services from more than one premises, in such cases he can apply for separate registration for each of such premises. He may apply for a single registration called Centralized Registration provided he has either centralized accounting or centralized billing system. In such cases one of the places shall be considered to be Head Office and all other premises shall be considered to be branches. A single registration certificate shall be issued. If the service provider do not have centralized accounting and also there is no centralized billing system, he will not be allowed to apply for centralized registration. Question 15: Write a note on Amendment of Registration Certificate. Answer: Amendment of Registration Certificate Rule 4(5A) of STR, 1994 A service provider may apply for amendment in registration in the following cases: 1. Change in place of business 2. Change in the name of business 3. Change in services rendered i.e. there may be addition / deletion 4. Any other similar change If there is any such change, service provider should apply to the department within 30 days for effecting the change. Question 16: Write a note on Cancellation of Registration Certificate. Answer: Cancellation of Registration Certificate Rule 4(7), 4(8) of STR, 1994 Every registered assessee, who ceases to provide the taxable service for which he is registered or if he dies, registration certificate shall be cancelled. The Superintendent of Central Excise shall ensure that the assessee has paid all monies due to the Central Government under the provisions of the Act, and the rules and the notifications issued thereunder, and thereupon cancel the registration certificate. Where a registered assessee transfers his business to another person, the transferee shall obtain a fresh certificate of registration.

Question 17: Write a note on issue of Invoice/Bill by Service Provider. Answer: As per Rule 4A of STR, 1994, every person providing taxable service, not later than fourteen days from the date of completion of such taxable service or receipt of any payment towards the value of such taxable service, whichever is earlier, shall issue an invoice, a bill or, signed by such person or a person authorized by him and such challan etc. shall be serially numbered and shall contain the following, namely: (i) the name, address and the registration number of such person; (ii) the name and address of the person receiving taxable service; (iii)description, classification and value of taxable service provided or to be provided; and (iv) the service tax payable thereon : Provided also that in case of continuous supply of service, every person providing such taxable service shall issue an invoice, bill or challan, as the case may be, within fourteen days of the date when each event specified in the contract, which requires the service receiver to make any payment to service provider, is completed: Every input service distributor distributing credit of taxable services shall, in respect of credit distributed, issue an invoice, a bill or, as the case may be, a challan signed by such person or a person authorized by him, for each of the recipient of the credit distri-buted, and such invoice, bill or, as the case may be, challan shall be serially numbered and shall contain the following, namely: (i) the name, address and registration number of the person providing input services and the serial number and date of invoice, bill, or as the case may be; (ii) the name and address of the said input service distributor; (iii) the name and address of the recipient of the credit distributed; (iv) the amount of the credit distributed : Question 18: Explain Determination of Rate for Service Tax. Answer: As per rule 5B of STR, 1994, the rate of tax in case of services provided, or to be provided, shall be the rate prevailing at the time when the services are deemed to have been provided under the rules made in this regard. Question 19 (V. Imp): Write a note on payment of Service Tax. Answer: Payment of Service Tax Section 68/ Rule 6 of STR, 1994 An individual or proprietary firm or partnership firm shall be required to pay service tax on quarterly basis and should be paid upto 5th of the month subsequent to the quarter in which the service is deemed to be provided (i.e. the quarter in which point of taxation falls) but if the payment is being made electronically through internet banking, payment can be made upto 6th of such month. However, payment for the last quarter should be made upto 31st March of that year. Any other service provider like Company or HUF etc. shall be required to pay service tax on monthly basis and it should be paid upto 5th of the month succeeding the month in which service is deemed to be provided but if payment is being made electronically through internet banking, payment can be made upto 6th of the month however, payment for last month should be made upto 31st March of that year

If an assessee has paid a total service tax of rupees ten lakh or more including the amount paid by utilization of Cenvat Credit, in the preceding financial year, he shall deposit the service tax liable to be paid by him electronically, through internet banking. As per section 77, if any person has failed to pay service tax electronically, he shall pay penalty which may extend upto `10,000. The assessee shall deposit the service tax liable to be paid by him in Form G.A.R-7 with the bank designated by the Central Board of Excise and Customs for this purpose. If the assessee deposits the service tax by cheque, the date of presentation of cheque to the bank designated by the Central Board of Excise and Customs for this purpose shall be deemed to be the date on which service tax has been paid subject to realization of that cheque. Rounding off of Duty, etc Section 37D (Central Excise Act, 1944) The amount of duty, interest, penalty, fine or any other sum payable, and the amount of refund or any other sum due, under the provisions of this Act shall be rounded off to the nearest rupee and, for this purpose, where such amount contains a part of a rupee consisting of paise then, if such part is fifty paise or more, it shall be increased to one rupee and if such part is less than fifty paise it shall be ignored. Question 20 (V. Imp.): Write a note on Point of Taxation for Payment of Service Tax. Answer: Point of Taxation for Payment of Service Tax Rule 3 Prior to 01.04.2011, service tax was to be paid on actual receipt basis but w.e.f. 01.04.2011, service tax shall be paid on the basis of point of taxation i.e. the point in time when the service shall be deemed to have been provided shall be taken into consideration. As per Rule 3, services shall be deemed to have been provided on the date of invoice issued by the service provider but if the invoice has been issued after expiry of 14 days from the date of completion of service, services shall be deemed to have been provided on the date of completion of service. If payment has been received before the date determined above, the date of providing the services shall be the date of receiving the payment and service tax should be paid accordingly. It can be further shown as given below: S. N o. 1. 2. Date of completion of service April 10, 2011 April 10, 2011 April 20, 2011 April 26, 2011 Date of invoice Date on which payment recd. April 30, 2011 April 30, 2011 Point of Taxation April 20, 2011 April 10, 2011 Invoice issued in 14 days and before receipt of payment Invoice not issued within 14 days and payment received Remarks

3. 4.

April 10, 2011 April 10, 2011

April 20, 2011 April 26, 2011

April 15, 2011 April 5, 2011 (part payment) and April 25, 2011 (balance payment) 15th April 2011

April 15, 2011 April 5, 2011 (part payment) and April 10, 2011 (balance payment) 01st April 2011

5.

April 10, 2011

1st April 2011

after completion of service Invoice issued in 14 days but payment received before invoice Invoice not issued in 14 days, point of taxation 10th April but for part payment received on 05th April 2011, POT shall be 05th April 2011. Invoice issued before completion of service

Service tax to be paid on actual receipt basis in certain cases Rule 7 As per rule 7 of point of taxation in the following cases, service tax shall be paid on actual receipt basis. 1. Individuals or proprietary firms or partnership firms providing taxable services referred below: (i) Consulting Engineer (ii) Practising Chartered Accountant (iii) Scientist or a Technocrat (iv) Legal Consultancy Services (v) Practising Company Secretary (vi) Practising Cost Accountant (vii) Interior Decorator (viii) Architect 2. In case of Reverse Charge, but payment should be made within a period of 6 months of the date of invoice otherwise point of taxation shall be determined in the normal manner. Question 21: Write a note on interest on delayed payment of Service Tax. Answer: Interest on delayed payment of service tax Section 75 If any person has failed to credit the tax to the Central Government within the prescribed time period, such person shall pay interest for the period of delay at the prescribed rate. As per notification no. 14/2011 dated 01.03.2011, the rate of interest shall be 18% p.a. Provided further that in the case of a service provider, whose value of taxable services provided in a financial year does not exceed sixty lakh rupees during any of the financial years covered by the notice or during the last preceding financial year, as the case may be, such rate of interest shall be reduced by three per cent per annum i.e. rate shall be 15% p.a. Question 22: Write a note on penalty for failure to pay Service Tax. Answer: Penalty for failure to pay service tax Section 76 If any person liable to pay service tax has failed to pay service tax, he shall be liable to pay service tax alongwith interest @ 18% p.a. and also penalty may be imposed which shall be `100 per day or 1% of such tax, per month whichever is higher starting with first day after the due date till the date of actual payment. However total amount of penalty shall not

exceed 50% of service tax payable. Example ABC Ltd, an assessee, fails to pay service tax of ten lakh rupees payable by 5 th March. ABC Ltd. pays amount on the 15th March. The default has continued for ten days The penalty payable by ABC Ltd. is computed as follows: 100 x 10 = `1,000 10,00,000 x 1% x 10/31 = `3,225.80 whichever is higher Penalty liable to be paid is ` 3226.00. Example Mr. X, an assessee, fails to pay service tax of `3,000 payable by 31st March. Mr. X pays the amount on 30th April. The default has continued for 30 days. The penalty payable by Mr. X is computed as follows: 100 x 30 = `3,000 3,000 x 1% x 30/30 = `30 whichever is higher. Penalty liable to be paid is `3,000 but penalty cannot exceed 50% of the amount of service tax, hence amount of penalty shall be `1,500 Question 23: Write a note on penalty for contravention of rules and provisions of Act for which no penalty is specified elsewhere. Answer: Penalty for contravention of rules and provisions of Act for which no penalty is specified elsewhere Section 77 (1) Any person who has failed to take registration shall pay penalty of `200 per day or `10,000 whichever is higher (2) Any person who has failed to maintain the book of accounts shall pay penalty which may extend upto `10,000. (3) Any person who has failed to furnish information or failed to furnish any document to the service tax officer shall pay penalty of `200 per day or `10,000 whichever is higher. (4) If any person has failed to pay service tax electronically shall pay penalty which may extend upto `10,000 (5) Any person, who contravenes any of the provisions of this Chapter or any rules made thereunder for which no penalty is separately provided in this Chapter, shall be liable to a penalty which may extend to ten thousand rupees. Question 24: Write a note on penalty for suppressing (to conceal) value of taxable services. Answer: Penalty for suppressing value of taxable service Section 78 If any person has not paid service tax or has short paid service tax or there was a refund of service tax because of fraud, wilful mis-statement etc., in such cases he shall be liable to pay service tax alongwith interest and also penalty shall be payable which shall be equal to the amount of service tax. Provided that where true and complete details of the transactions are available in the specified records, penalty shall be reduced to 50% of the service tax so not levied or paid or short-levied or short-paid or erroneously (wrongly) refunded. If

the tax and interest has been paid within 30 days of receiving the orders from the Central Excise Officer, penalty payable shall be 25% of such service tax. Further such penalty should also be paid within the period of 30 days as mentioned above. If the service provider whose value of taxable services do not exceeds `60 lakhs during any of the year covered by the notice or during the last preceding financial year, the period of 30 days shall be taken as 90 days. Provided also that if the penalty is payable under this section, the provisions of section 76 shall not apply. Question 25 (Imp.): Write a note on Advance Payment of Service Tax. Answer: Advance payment of Service Tax Rule 6 of STR, 1994 Every person liable to pay service tax, may, on his own volition (voluntary), pay an amount as service tax in advance, to the credit of the Central Government and adjust the amount so paid against the service tax which he is liable to pay for the subsequent period : Provided that the assessee shall, (i)intimate the details of the amount of service tax paid in advance, to the jurisdictional Superintendent of Central Excise within a period of fifteen days from the date of such payment; and (ii) indicate the details of the advance payment made, and its adjustment, if any in the subsequent return to be filed under section 70 of the Act. Question 26 (Imp.): Write a note on Refund of Service Tax/ Issue of Credit Note. Answer: Refund of Service Tax / Issue of Credit Note Rule 6 of STR, 1994 Where an assessee has issued an invoice, or received any payment, against a service to be provided which is not so provided by him either wholly or partially for any reason or where the amount of invoice is renegotiated due to deficient provision of service, or any terms contained in a contract, the assessee may take the credit of such excess service tax paid by him, if the assessee, (a) has refunded the payment or part thereof, so received for the service provided to the person from whom it was received; or (b)has issued a credit note for the value of the service not so provided to the person to whom such an invoice had been issued. Question 27 (Imp.): Write a note on Provisional Payment of Service Tax. Answer: Provisional payment of Service Tax Rule 6 of STR, 1994 In case the assessee is unable to correctly estimate, at the time of the deposit, the actual amount of service tax for any month or quarter, he may make a written request to Assistant/ Deputy Commissioner of Central Excise for making payment of service tax on provisional basis. The concerned officer may allow payment of service tax on provisional basis on such value of taxable service as may be specified by him. For the purpose of provisional assessment at the time of filing the return, the assessee is required to file a statement in form ST - 3A giving detail of difference between service tax deposited and the service tax liable to be paid for each month. The Assistant/Deputy Commissioner of Central Excise, on the basis of memorandum in form ST - 3A may complete the assessment after calling for necessary documents or records, if need be.

Question 28 (V. Imp.): Write a note on Self Adjustment of Service Tax. Answer: Self adjustment of Service Tax Rule 6 of STR, 1994 Where an assessee has paid to the credit of Central Government any amount in excess of the amount required to be paid towards service tax liability for a month or quarter, as the case may be, the assessee may adjust such excess amount paid by him against his service tax liability for the succeeding month or quarter, as the case may be. The adjustment of excess amount paid, shall be subject to the following conditions, namely: (i)excess amount paid is on account of reasons not involving interpretation of law, taxability, classification, valuation or applicability of any exemption notification, (ii)excess amount paid by an assessee having centralised registration, on account of delayed receipt of details of payments towards taxable services may be adjusted without monetary limit, (iii)in cases other than specified in clause (ii) above, the excess amount paid may be adjusted with a monetary limit of two lakh rupees for a relevant month or quarter, as the case may be, (iv)the details and reasons for such adjustment shall be intimated to the jurisdictional Superintendent of Central Excise within a period of fifteen days from the date of such adjustment. As per section 11B of Central Excise Act, 1944 if excess amount paid is on account of reasons involving interpretation of law, taxability, classification, valuation or applicability of any exemption notification, in such cases, the service provider should claim refund and for this purpose he should make an application before expiry of one year from the date of making the payment to Central Excise Officer and he shall grant him refund provided incidence of service tax has not been passed on to some other person otherwise amount shall be credited to Consumer Welfare Fund. As per section 11BB if refund has not been granted within 3 months from the date of receipt of the application, interest shall be allowed @ 6% p.a. from the date immediately after expiry of the period of 3 months till the date of refund (Notification No. 67/2003 dated: 12.09.2003) Question 29 (Imp.): Explain payment of service tax in case of Associated Enterprises. Answer: The term associated enterprises is not defined under service tax provision rather it is defined under section 92A of Income Tax Act and it means an enterprise (a) which participates, directly or indirectly, or through one or more intermediaries, in the management or control or capital of the other enterprise; or (b) in respect of which one or more persons who participate, directly or indirectly, or through one or more intermediaries, in its management or control or capital, are the same persons who participate, directly or indirectly, or through one or more intermediaries, in the management or control or capital of the other enterprise.

Example Telco manufactures Mercedes cars in India using the know how of Mercedes Germany. Mercedes cars cannot be manufactured in India without such know how and therefore Telco & Mercedes Germany are associated Enterprises. Example Unilever U.K. holds 26% equity shares of Hindustan Lever. Therefore Unilever & Hindustan Lever are associated Enterprises. Point of Taxation As per Rule 7 of Point of Taxation Rule, in case of associated enterprises, where the person providing the service is located outside India, the point of taxation shall be the date of credit in the books of account of the person receiving the service or date of making the payment whichever is earlier. Question 30: Write a note on adjustment for Municipal Tax paid for House Property. Answer: Adjustment for Municipal Tax paid for house property Rule 6 of STR, 1994 Where the person liable to pay service tax in respect of renting of immovable property, has paid any amount in excess of the amount required to be paid towards service tax liability on account of non-availment of deduction of property tax paid, from the gross amount charged for renting of the immovable property, the assessee may adjust such excess amount paid by him against his service tax liability within one year from the date of payment of such property tax. The details of such adjustment shall be intimated to the Superintendent of Central Excise within a period of fifteen days from the date of such adjustment. Example X Ltd. Gives a commercial property on rent to Y Ltd. (monthly rent being `2,00,000) Y Ltd. Pays rent on the last day of each month. Municipal tax for the financial year 2011-12 is Rs.4,44,000 (due date of payment is June 30,2011, whereas it is actually paid on September 30, 2011 along with late payment interest of Rs.16,200) Value of taxable service for the financial Year 2011-12 will be as follows: Financial Year 2011-12 April May June July August September October November December January February March Gross rent ` 2,00,000 2,00,000 2,00,000 2,00,000 2,00,000 2,00,000 2,00,000 2,00,000 2,00,000 2,00,000 2,00,000 2,00,000 Municipal tax (`4,44,000 12 = `37,000) ` 1,85,000 + 37,000 = 2,22,000 37,000 + 22,000 = 59,000 37,000 37,000 37,000 37,000 37,000 Taxable Value of service ` 2,00,000 2,00,000 2,00,000 2,00,000 2,00,000 (-) 22,000 1,41,000 1,63,000 1,63,000 1,63,000 1,63,000 1,63,000

Municipal tax is paid on September 30, 2011. Municipal tax liability from April 2011 till August 2011 comes to Rs.1,85,000 (i.e., `37,000 5). This unadjusted amount can be adjusted at any time within 1Year from the date of payment of municipal tax. The details of such adjustment should be informed to jurisdictional Superintendent of Central Excise within period of 15 days from date of adjustment [rule 6(4c) of Service Tax Rules] Question 31 (Imp.): Write a note on service tax collected wrongly or in excess from any person. Answer: As per section 73A, if any person has collected any service tax in excess or wrongly from the recipient of taxable service in any manner, in that case such person should deposit such amount with the Central Government. Central Government shall refund the amount to the person from whom tax has been collected and for this purpose Central Government may issue a public notice and such person can make an application within 6 months from the date of public notice. If the amount has not been claimed by any person, it will be deposited in the Consumer Welfare Fund which is regulated by a committee and its members are nominated by the Government. The amount is utilized for the welfare of consumer in India. As per section 73B, such person shall also pay interest @ 18% p.a. from the first day of the month succeeding the month in which the amount ought to have been paid, till the date of payment of such amount. However if the total value of taxable services in the relevant financial year or in the preceding year is not exceeding `60,00,000, rate of interest shall be 15% p.a. Question 32 (Imp.): Write a note on Point of Taxation for determining the rate of Service Tax. Answer: Point of taxation for determining the rate of Service Tax Rule 4 of POT Rules, 2011 Rate of service tax shall be determined as per Rule 4 in the manner given below: Case 1: Taxable services have been completed before the change in effective rate of tax Situation where the invoice issued as well as the payment received after the change in effective rate of tax where the invoice is issued prior to change in effective rate of tax but the payment is received after the change in effective rate of tax where the payment is received before the change in effective rate of tax, but the invoice for the same has been issued after the change in effective rate of tax Date of Receipt Point of Taxation Date of Receipt or issuing of invoice, whichever is earlier Date Of Issue Of Invoice

It will be further clear with help of the following illustrations. S. No . 1. 2. 3. Date of completion of service April 10, 2011 April 10, 2011 April 10, 2011 Change in effective rate of tax April 25, 2011 April 25, 2011 April 25, 2011 April 26, 2011 April 15, 2011 April 30, 2011 April 30, 2011 April 30, 2011 April 15, 2011 April 26, 2011 April 15, 2011 April 15, 2011 New rate Old rate Old rate Date of invoice Date on which payment recd. Point of Taxation Rate applicable

Case 2: Services have been completed after the change in effective rate of tax Situation change in effective rate of tax but the invoice has been issued prior to the change in effective rate of tax where the invoice has been issued and the payment date of receipt of payment or date of issuance of for the invoice received before the change in invoice, whichever is earlier effective rate of tax where the invoice has also been raised after the date of issuing of invoice change in effective rate of tax but the payment has been received before the change in effective rate of tax The applicability of the rule will be clear from the illustrations in the following table: S. N o. 1. 2. 3. Date of completion of service April 30, 2011 April 30, 2011 April 30, 2011 Change in effective rate of tax April 25, 2011 April 25, 2011 April 25, 2011 Date of invoice April 20, 2011 April 20, 2011 April 30, 2011 Date on which payment recd. April 30, 2011 April 15, 2011 April 20, 2011 Point of Taxation April 30, 2011 April 15, 2011 April 30, 2011 Rate of Tax New rate Old rate New rate Point of Taxation

where the payment for the invoice is made after the date of Receipt of payment

Question 33: Write a note on liability to service tax in case of New Services. Answer: New Services Rule 5 of POT Rules, 2011 As per Rule 5, in case of new services, no service tax shall be charged if invoice has been issued and also payment has been received before such service became taxable. If payment has been received before the service became taxable and also invoice has been issued within 14 days, no service tax shall be charged. Example

Hotel accommodation services have become taxable w.e.f. 01.05.2011 and Ashoka Hotel has received ` 12,00,000 from ABC Ltd. in connection with stay of their employees for one month from 01.05.2011 to 30.06.2011. Full payment was received on 18.04.2011 and invoice was issued on the same date, in this case, no service tax is payable. If in this case, payment was received on 27.04.2011 and invoice was issued on 07.05.2011, even then services are exempt. If payment was received on 01.05.2011 and invoice was issued on 03.05.2011, in this case, service tax shall be paid however, exemption shall be allowed for `10,00,000 and service tax shall be charged on the balance amount.

Question 34: What are the provisions of service tax for Continuous Supply of Service. Answer: Continuous Supply of Service Rule 6 of POT Rules, 2011 As per Rule 2 Continuous Supply of service means any service which is provided, or to be provided continuously, under a contract, for a period exceeding three months, or where the Central Government, by a notification specify such service to be a continuous supply of service, whether or not subject to any condition. As per Rule 6 of point of taxation for continuous supply of service shall be determined as below Situation Point of Taxation For the service provided or to be provided The time when the invoice is issued Where the invoice is not issued within fourteen days of the Date of completion of Service completion of the provision of the service If Advance is Received Date of receipt of each such advance

Where the provision of the whole or part of the service is determined periodically on the completion of an event in terms of a contract, which requires the service receiver to make any payment to service provider, the date of completion of each such event as specified in the contract shall be deemed to be the date of completion of provision of service. For example, in the case of construction services if the payments are linked to stage-by-stage completion of construction, the provision of service shall be deemed to be completed in part when each such stage of construction is completed. The following services have been notified as continuous supply of services. (a) Telecommunication service [65(105)(zzzx)] (b) Commercial or industrial construction [65(105)(zzq)] (c) Construction of residential complex [65(105)(zzzh)] (d) Internet Telecommunication Service [65(105)(zzzu)] (e) Works contract service [65(105)(zzzza)] Thus these services will constitute continuous supply of services irrespective of the period for which they are provided or agreed to be provided. Other services will be considered continuous supply only if they are provided or agreed to be provided continuously for a period exceeding three months. Illustration 1: ABC Ltd. is a construction company and it has entered into an agreement with XYZ Ltd. to construct one

building which will have 4 floor and as per the contract. Each floor shall be considered to be one stage of completion. ABC Ltd. submitted particulars as given below: Completed Ground Floor Issued Bill Half payment of ` 10,00,000 plus service tax received And balance payment of ` 10,00,000 plus Service Tax received Completed 1st floor Issued Bill Half payment of ` 9,00,000 plus service tax received And balance payment of ` 9,00,000 plus Service Tax received Completed 2nd Floor Issued Bill Half payment of ` 8,00,000 plus service tax received And balance payment of ` 8,00,000 plus Service Tax received Completed 3rd Floor Issued Bill Full amount of ` 10,00,000 plus Service Tax received on 16/06/2011 on 01/07/2011 on 27/06/2011 on 10/07/2011 on 27/08/2011 on 07/09/2011 on 31/08/2011 on 10/09/2011 on 30/11/2011 on 10/11/2011 on 30/10/2011 on 01/12/2011 on 10/03/2012 on 01/04/2012 on 01/08/2012

In FY 10-11, Bill issued by company were ` 200 lakh and service tax was paid @ 10.3 % Payment of service tax for March 2012 was made on 21/04/2012 Show tax treatment of service tax for each month and also compute interest and penalty payable u/s 75 and 76 respectively. Solution: ` April 2011 May 2011 June 2011 Taxable value Service tax @10.3% July 2011 Aug 2011 Taxable value Service tax @ 10.3% Sept 2011 NIL NIL 20,00,000 20,6000 NIL 9,00,000 92,700

Taxable value Service tax @ 10.3 % Oct 2011 Taxable value Service tax @ 10.3 % Nov 2011 Taxable value Service tax @ 10.3 % Dec 2011 Jan, Feb 2012 March 2012 Taxable value Service tax @10.3% Delay in payment 21 days Interest u/s 75 ` 1,03,000 x 18% x 21/365 = 1066.68 Penalty u/s 76 ` 1,03,000 x 1% x 21/30 = ` 721 OR 21 x 100 = ` 2100 Whichever is higher Thus penalty will be

9,00,000 92,700 8,00,000 82,400 8,00,000 82,400 NIL NIL 10,00,000 1,03,000 1,067

2,100

Question 35: Write a note on determination of Point of Taxation in case of Copyrights, etc. (For Self Reading) Answer: Determination of Point of Taxation in case of Copyrights, etc. Rule 8 In respect of royalties and payments pertaining to copyrights, trademarks, designs or patents, where the whole amount of the consideration for the provision of service is not ascertainable at the time when service was performed, and subsequently the use or the benefit of these services by a person other than the provider gives rise to any payment of consideration, the service shall be treated as having been provided each time when a payment in respect of such use or the benefit is received by the provider in respect thereof, or an invoice is issued by the provider, whichever is earlier. Question 36(V. Imp.): Write a note on Filing of Service Tax Return. Answer: Furnishing of Returns Section 70 / Rule 7 of STR, 1994 Every assessee liable to pay service tax shall submit a half yearly return in From ST-3 or ST-3A, as the case may be, along with a copy of the Form GAR-7 for the months / quarters covered in the half-yearly return. Every assessee shall submit the half yearly return by the 25th of the month following the particular half-year. If the last date is a holiday, return can be filed on the next day without any penalty.

Every assessee shall submit the half-yearly return electronically. Every service provider who is registered shall also be required to file the return even if service tax liability is nil i.e. if any service provider has gross receipt exceeding `9,00,000 shall be required to get registered and in that case return has to be filed even if service tax liability is nil. Every assessee shall furnish to the Superintendent of Central Excise at the time of filing his return for the first time a list of all accounts maintained by the assessee in relation to service tax including memoranda received from his branch offices. Question 37: Write a note on Revision of Service Tax Return. Answer: Revision of Return Rule 7B of STR, 1994 An assessee may submit a revised return, in Form ST-3, in triplicate, to correct a mistake or omission, within a period of 90 days from the date of submission of the return under rule 7. Question 38 (V. Imp.): Write a note on Delay Furnishing of Service Tax Return. Answer: Amount to be paid for Delay in Furnishing Return Rule 7C of STR, 1994 Where the return prescribed under rule 7 is furnished after the date prescribed for submission of such return, the person liable to furnish the said return shall pay to the credit of the Central Government, for the period of delay of(i) 15 days from the date prescribed for submission of such return, an amount of `500. (ii) beyond 15 days but not later than 30 days from the date prescribed for submission of such return, an amount of `1,000; and (iii) beyond 30 days from the date prescribed for submission of such return an amount of `1,000 plus `100 for every day from the thirty first day till the date of furnishing the said return. Provided that the total amount payable in terms of this rule, for delayed submission of return, shall not exceed `20,000. Provided also that where the gross amount of service tax payable is nil, the Central Excise Officer may, on being satisfied that there is sufficient reason for not filing the return, reduce or waive the penalty. Question 39: Write a note on contents of the Service Tax Return. Answer: Contents of the Return 1. Half year for which return is being filed 2. Name of the assessee 3. STC Number 4. Address 5. Constitution of the assessee like individual or partnership firm etc. 6. Assessee liable to pay service tax as service provider or service receiver under reverse charge 7. Gross amount received 8. Amount of service tax and education cess 9. Tax credit allowed 10. Net tax payable 11. Details of services rendered 12. Verification of the return

13. Signature of service provider or service recipient under reverse charge Question 40: Write a note on documents to be submitted along with Service Tax Return. Answer: Documents to be submitted along with the return Alongwith ST-3 return following documents should be attached: (i) copies of GAR-7 challans which indicate the payment of service tax. (ii) a memorandum in form ST-3A giving details of the difference between the amount of provisional tax deposited and the actual amount payable for each month. Every service tax assessee should furnish to the Superintendent of Central Excise, at the time of filing first half yearly return, a list of all accounts maintained by him in relation to service tax. The service tax return should be filed in triplicate to the Superintendent of Central Excise . The assessee may also file the return with the concerned Divisional Office by registered post However, the assesses should ensure that the return reaches the Divisional Office on or before the due date. For an assessee who provides more than one taxable service, filing of a single return is sufficient. However, the details in each of the columns of the Form ST-3 have to be furnished separately for each of the taxable service rendered by him. Even if no service has been provided during a half year and no service tax is payable; the assessee has to file a Nil return within the prescribed time limit. Question 41 (V. Imp.): Write a note on E-Filing of Service Tax Return. Answer: E-Filing of Return With effect from 01.10.2011, e-filing of service tax returns has been made mandatory for all the assesses (Notification No. 43/2011 dated: 25.08.2011). The assessee can e-file the return through software ACES i.e. AUTOMATION OF CENTRAL EXCISE AND SERVICE TAX. The assessee should login the relevant site www.aces.gov.in. To transact business on ACES a user has to first register himself with ACES through a process called Registration with ACES. The assessee should have STP code i.e. PAN based 15 digit registration number allotted by Service Tax Department. The assessee should fill in self-chosen user-ID and his email ID and system will check the availability of the chosen user ID and then generate a password and will send it to the assessee on the e-mail ID mentioned by him and the assessee can submit the return in the prescribed form. Benefits of E-Filing for assesee: 1) Reduce Physical Interface with the Department; 2) Save Time; 3) Reduce Paper Work; 4) Online Registration and Amendment of Registration; 5) Electronic filing of all documents such as applications for registration, returns, claims, permissions and intimations; export-related documents, refund request; 6) System-generated E-Acknowledgement; 7) Online tracking of the status of selected documents. Question 42: Write a note on Service Tax Return Preparer Scheme. Answer: Service Tax Return Preparer Scheme Section 71 Department shall appoint Service Tax Return Preparer to assist the persons to file service tax return. The STRP shall have

required qualification i.e. he should have passed senior secondary level examination. The Service Tax Return Preparer shall (a) (b) (c) (d) (e) prepare the return with due diligence; affix his signature on the return prepared by him; furnish the return as specified; hand over a copy of the return to the person whose return is prepared and furnished by him; retain a copy of the acknowledgment of having furnished the return

In respect of returns prepared and furnished by him maintain record of the following, namely:(i) the name of assessees whose returns have been prepared and furnished by him during that month; (ii) the Service Tax Code(STC) number and premises code of such assessees; (iii) period for which return is filed; (iv) date of furnishing the return; (v) authority with whom return is filed; (vi) amount of tax payable; (vii) amount of tax paid; (viii) the fee charged and received by him Incentive to Service Tax Return Preparers An assessee shall pay a fee as may be mutually agreed upon between an assessee and the Service Tax Return Preparer. The Board recommends, as a yardstick, a fee of rupees one thousand rupees per return prepared by the Service Tax Return Preparer.

Question 43: Write a note on Records under Service Tax. Answer: Records Rule 5 of STR, 1994 The records including computerised data, as maintained by an assessee in accordance with the various laws in force from time to time shall be acceptable. All such records shall be preserved at least for a period of 5 years immediately after the financial year to which such records pertain. Every assessee shall make available, at the registered premises, at all reasonable time, all such records, for inspection and examination by the Central Excise Officer authorised in writing by the jurisdictional Assistant Commissioner or Deputy Commissioner of Central Excise, as the case may be. Explanation. - For the purposes of this rule, registered premises includes all premises or offices from where an assessee is providing taxable services. As per section 77, any person who has failed to maintain the book of accounts shall pay penalty which may extend upto `10,000. Question 44 (V. Imp.): Write a note on option to pay service tax in case of Air Travel Agent. Answer: Option to pay service tax in case of Air Travel Agent Rule 6(7) As per section 65(105)(l) - Taxable service means any service provided or to be provided, to any person, by an Air Travel Agent in relation to the booking of passage for travel by air.

As per section 65(4) -Air Travel Agent means any person engaged in providing any service connected with the booking of passage for travel by air. The person liable for paying the service tax in relation to the services provided by an air travel agent, shall have the option, to pay an amount at the rate of 0.6% plus EC of the basic fare in the case of domestic bookings, and at the rate of 1.2% plus EC of the basic fare in the case of international bookings, of passage for travel by air, during any calendar month or quarter, as the case may be, towards the discharge of his service tax liability instead of paying service tax at the normal rate and the option, once exercised, shall apply uniformly in respect of all the bookings of passage for travel by air made by him and shall not be changed during a financial year under any circumstances. Basic Fare means that part of the air fare on which commission is normally paid to the air travel agent by the airline. Illustration 2: Mr. Anurag is an Air Travel Agent and provides the following information for the month of October, 2011. Date Particulars Basic Fare Taxes Other Commission Total Value than Service tax 2/10/2011 Booked by Mr. A for 3,000 2,500 500 6,000 Delhi to Chennai 10/10/2011 Booked by Mr. Somnath 15,000 2,500 1,500 19,000 for Delhi to Bangkok 24/10/2011 Booked by Mr. Raj for 2,600 2,400 500 5,500 Delhi to Bhubaneswar Compute service tax payable by Mr. Anurag under various methods applicable for air travel agent. Solution: Method 1 Total Commission Service tax @ 10.3% Rounded off u/s 37D Method 2 As per Rule 6(7) of Service Tax Rules 1994 02.10.2011 3,000 x 0.618% 10.10.2011 15,000 x 1.236% 24.10.2011 2,600 x 0.618% Total Service Tax Payable Rounded off u/s 37D ` 18.54 185.40 16.07 220.01 220.00

2,500.00 257.50 258.00

Question 45 (V. Imp.): Write a note on option to pay service tax by any insurer carrying on Life Insurance Business. Answer: Option to pay service tax by any insurer carrying on life insurance business Rule 6(7A) As per section 65(105)(zx) - Taxable service means any service provided or to be provided, to a policy holder or any

person, by an insurer, including re-insurer carrying on life insurance business. An insurer carrying on life insurance business shall have the option to pay tax: (i) on the gross premium charged from a policy holder reduced by the amount allocated for investment, or savings on behalf of policy holder, if such amount is intimated to the policy holder at the time of providing of service; (ii) 1.5 per cent plus EC of the gross amount of premium charged from a policy holder in all other cases; towards the discharge of his service tax liability instead of paying service tax at the normal rate. Provided that such option shall not be available in cases where the entire premium paid by the policy holder is only towards risk cover in life insurance. Illustration 3: Mr. X is taken a policy from LIC and premium payable is `50,000 per annum for 20 years (including Risk Premium of `10,000). Calculate the service tax to be charged by LIC under various methods. Solution: Method 1 Risk Premium x service tax rate 10,000 x 10.3% = `1,030 Method 2 Gross Amount charged x 1.545 % 50,000 x 1.545% = `772.5 Rounded off U/s 37D = ` 773 Question 46 (Imp.): Write a note on option to pay service tax in case of Money Changer. Answer: Option to pay service tax in case of a Money Changer Rule 6(7B) As per section 65(105)(zzk) - Money Changer service means any service provided or to be provided, to any person, by a Foreign Exchange Broker, including an authorised dealer in foreign exchange or an authorised money changer. The person liable to pay service tax in relation to purchase or sale of foreign currency, including money changing, provided by a foreign exchange broker, shall have the option to pay an amount calculated at the following rate towards discharge of his service tax liability instead of paying service tax at the normal rate: (a) 0.1% of the gross amount of currency exchanged upto ` 100,000, but minimum ` 25; (b) on next 9,00,000 @ 0.05% (c) on balance @ 0.01% but maximum `5,000. Provided that the person providing the service shall exercise such option for a financial year and such option shall not be withdrawn during the remaining part of that financial year. Alternatively, the assessee can find value of service as provided in rule 2B of Service Tax (Determination of Value) Rules, 2006.

As per these rules, the value of service will be equal to difference in buying or selling rate (as the case may be) and the RBI reference rate for the currency at that time. [RBI Reference Rate is mean of selling and buying rate]. Example I: US$1000 are sold by a customer at the rate of Rupees 45 per US$. RBI reference rate for US$ is Rupees 45.50 for that day. The taxable value shall be Rupees 500. Example II: INR70000 is changed into Great Britain Pound (GBP) and the exchange rate offered is Rupees 70, thereby giving GBP 1000. RBI reference rate for that day for GBP is Rupees 69. The taxable value shall be Rupees 1000. Illustration 4: Mr. X is a dealer and engaged in sale & purchase of Foreign currency. ABC Ltd requires 10000 US Dollar to make foreign payment. Mr. X quotes ` 51 per US Dollar and RBI reference rate is `50 per dollar. Calculate the amount of service tax under different option: Solution: Option 1 10,000 (51-50) x 10.3% Option 2 Gross Amount Charged 51 x 10,000 Service Tax Up to 1,00,000 Next 4,10,000 Education cess @ 3% Service tax payable Rounded Off u/s 37D ` 1,030 5,10,000

1,00,000 x 0.1% 4,10,000 x 0.05%

100.00 205.00 305.00 9.15 314.15 314.00

Question 47: Write a note on option to pay service tax in case of Distribution of Lottery ticket. Answer: Option to pay service tax in case of distributor of lottery tickets Rule 6(7C) As per section 65(105)(zzzzn) -Distribution of lottery ticket service means any service provided or to be provided to any person, by any other person, for promotion, marketing, organising or in any other manner assisting in organising games of chance, including lottery, Bingo or Lotto in whatever form or by whatever name called, whether or not conducted through internet or other electronic networks. The distributor or selling agent, liable to pay service tax for the taxable service of promotion, marketing, organising or in

any other manner assisting in organising lottery, referred to in sub-clause ( zzzzn) of clause (105) of section 65 of the said Act, shall have the option to pay an amount at the rate mentioned below: Sl. No . (1) 1. Rate Condition

(2) ` 6,000 on every ` 10 lakh (or part of ` 10 lakh) of aggregate face value of lottery tickets printed by the organising State for a draw ` 9,000 on every ` 10 lakh (or part of ` 10 lakh) of aggregate face value of lottery tickets printed by the organising State for a draw

(3) If the lottery or lottery scheme is one where the guaranteed prize payout is more than 80% If the lottery or lottery scheme is one where the guaranteed prize payout is less than 80% :

2.

In case of online lottery, the aggregate face value of lottery tickets shall be taken as the aggregate value of tickets sold. The distributor or selling agent shall exercise such option within a period of one month of the beginning of each financial year and such option shall not be withdrawn during the remaining part of the financial year. Illustration 5: M/s Future Gaming Solutions India Private Limited is a distributor of lottery organized by State of Sikkim provides following information: Particulars Diwali Bumper Diwali Dhamaka Total No of Tickets 2,50,000 2,50,000 Face Value of Tickets 100 100 Value of Guaranteed Prize Payout 51.11% 51.11% Actual Number of ticket sold 2,00,000 2,00,000 Mode of conducting the scheme Printed On line Compute Service tax payable under Rule 6(7C) of service tax Rules, 1994. Solution: ` Printed Mode 2,50,000 x 100 x 9,000 = 10,00,000 Education Cess @ 3% On Line Mode 2,00,000 x 100 x 9,000 10,00,000 Education Cess @ 3% 2,25,000 6,750 2,31,750 1,80,000 5,400 1,85,400

Illustration 6: Mr. Amit Rastogi is a distributor of lotteries organized by the State of Haryana. He is running two schemes of lotteries as followedTotal No. of ticket proposed under the scheme Face value per ticket Value of guaranteed prize payouts Actual no. of tickets sold Mode of conducting the Scheme Scheme A 22,75,000 10 75% 17,50,000 Printed 22,75,000 Scheme B 50,000 250 85% 46,250 Online

Compute service tax payable under Rule 6(7C) of Service Tax Rule, 1994. Solution: Computation of Service Tax Liability of Mr. Amit Rastogi Statement showing computation of Service Tax liability under Composition Scheme (Rule 6(7C) of the STR, 1994) of Mr. Amit Rastogi. Particulars Scheme A Scheme B Total no. of tickets 22,75,000 46,250 Face value per ticket 10 250 Aggregate value of lottery tickets 2,27,50,000 1,15,62,500 Value of guaranteed prize payouts 75% 85% No. of units of 10 lakhs or part thereof 23 12 Service tax payable(for every 10 lakhs or part thereof) 9,000 6,000 Service tax payable 2,07,000 72,000 Total service tax 2,79,000 Add: EC @ 2% 5,580 Add: SHEC @ 1% 2,790 Total service tax liability 2,87,370 Question 48 (V. Imp.): Write a note on Reverse Charge Mechanism. Answer: Reverse Charge Mechanism Section 68(2)/ Rule 2(1)(d) In general service provider has to charge service tax from the service recipient and has to pay to the government but in some of the services, service recipient shall be liable to pay service tax directly to the government and it is called reverse charge mechanism and it is applicable in the following cases: 1. Insurance Auxiliary Service In relation to insurance auxiliary service by an insurance agent, any person carrying on the general insurance business or the life insurance business, as the case may be, in India. 2. Import of Services In relation to any taxable service provided or to be provided by any person from a country other than India and received by any person in India under section 66A of the Act, the recipient of such service (import of services).

3. Goods Transport Agency In case of transportation of goods by goods transport agency, service tax shall be paid by the consignor or the consignee whosoever is liable to pay freight charges under Reverse Charge but Reverse Charge is applicable only when consignor or consignee is one of the persons mentioned below otherwise Reverse Charge is not applicable. (a)any factory registered under or governed by the Factories Act, 1948; (b)any company formed or registered under the Companies Act, 1956; (c)any corporation established by or under any law; (d)any society registered under the Societies Registration Act, 1860 (e)any co-operative society established by or under any law; (f) any dealer of excisable goods, who is registered under the Central Excise Act, 1944 or the rules made thereunder; or (g) any body corporate established, or a partnership firm registered, by or under any law, 4. Business Auxiliary Service of Distribution of Mutual Fund In relation to business auxiliary service of distribution of mutual fund by a mutual fund distributor or an agent, as the case may be, the mutual fund or asset management company, as the case may be, receiving such service. 5. Sponsorship Service In relation to sponsorship service provided to any body corporate or firm located in India, the body corporate or, as the case may be the firm who receives such sponsorship service shall be liable to pay service tax but if the person sponsoring the programme is located outside India, Reverse Charge shall not be applicable. The above persons shall apply for registration as if they are service provider and all the provisions of section 69 and rule 4 shall apply but general exemption as per notification no 6/2005 shall not apply. The service recipient shall be allowed Tax credit for the service tax paid by him under reverse charge mechanism, however it cannot be paid through Tax credit rather payment should be made and only after that its CENVAT credit shall be allowed. As per Rule 7 of point of Taxation Rule, 2011, in case of reverse charge point of taxation shall be the date on which actual payment has been made but if such payment is not made within 6 months of the date of invoice, in that case point of taxation shall be determined in the normal manner. Question 49 (V. Imp.): Write a note on Insurance Auxiliary (help) Service. Answer: In relation to insurance auxiliary service provided by an insurance agent, the person liable to pay service tax is the person carrying on general insurance business or the life insurance business as the case may be, in India. Example If LIC has to pay commission of `2,00,000 to their agent Mr. X, in this case LIC is the service recipient but still LIC has to pay service tax under reverse charge and service tax payable shall be `2,00,000 x 10.3% = 20,600 and amount of TDS under section 194D shall be 2,00,000 x 10% = `20,000

Question 50 (V. Imp.): Write a note on Import of Services. Answer: Section 66A/ Taxation of services (provided from outside India and received in India) Rules, 2006 Where any taxable service have been provided by any person from outside India to any person in India in such cases the person who has taken services from outside India shall be liable to pay service tax as if he is the service provider and he should apply for registration even if the gross amount is less than `9 lakh and also he should pay service tax even if the gross amount is not exceeding `10 lakhs and it is called Reverse Charge Mechanism. If the service recipient is an individual and services are not taken for business or commerce, in that case, he is not liable to pay service tax. Question 51 (V. Imp.): Write a note on Goods Transport Agency. Answer: As per section 65(105)(zzp) - Goods Transport Agency Service means any service provided or to be provided, to any person, by a goods transport agency, in relation to transport of goods by road in a goods carriage. As per section 65(50b) - Goods Transport Agency means any person who provides service in relation to transport of goods by road. In relation to taxable service provided by a goods transport agency, Reverse Charge shall be applicable and service tax shall be paid by the consignor or consignee whosoever is liable to pay freight (transportation charges) and Reverse Charge shall be applicable where the consignor or consignee of goods is atleast one of the persons mentioned below: (a) Any factory registered under by the Factories Act, 1948; (b) Any company registered under the Companies Act, 1956; (c) Any corporation established under any law; (d) Any society registered under the Societies Registration Act, 1860; (e) Any co-operative society; (f) Any dealer of excisable goods, who is registered under the Central Excise Act, 1944; (g) Any body corporate established, under any law (h) Any partnership firm registered. If neither the consignor nor the consignee is one of the persons mentioned above, in that case service tax shall be paid by the Goods Transport Agency and Reverse Charge shall not be applicable. Example If ABC Ltd. of Delhi has taken services of GTA and goods have been sent to Bombay and freight is payable by the consignee at Bombay, in this case service tax shall be paid by the consignee and not by the GTA. Question 52: Write a note on Business Auxiliary Services of Distribution of Mutual Fund. Answer: In relation to Business Auxiliary Service of Distribution of Mutual Fund by a mutual fund distributor or an agent, as the case be, the person liable for paying service tax is the mutual fund or asset management company, as the case may be, receiving such service.

Question 53 (V. Imp.): Write a note on Sponsorship Services. Answer: As per section 65(99a) - Sponsorship includes naming an event after the sponsor, displaying the sponsors company logo or trading name, giving the sponsor exclusive or priority booking rights, sponsoring prizes or trophies for competition; but does not include any financial or other support in the form of donations or gifts, given by the donors subject to the condition that the service provider is under no obligation to provide anything in return to such donors. In relation to sponsorship service provided to any body corporate or firm located in India, the person liable to pay service tax is the body corporate or firm, as the case may be who receives such sponsorship service. If the person sponsoring the event is out of India, in that case, reverse charge shall not be applicable rather the person organizing the event i.e. the service provider shall be liable to pay service tax. Question 54 (V. Imp): Write a note on tax credit in case of Service Provider. Answer: A Service provider shall be allowed tax credit for the following tax paid by him. 1. Excise duty on inputs (raw material etc) or capital goods (plant and machinery, furniture and fixtures etc) used in or in connection with providing of output services. 2. Service Tax paid on the input services taken by him in connection with providing of output services and such input services may be insurance services, banking services, renting of immovable property or other similar services. 3. Education cess paid on Excise Duty or Countervailing Duty or Service Tax shall also be eligible for tax credit. The service provider can utilize the tax credit in the manner given below: 1. Tax credit for Excise Duty or Service Tax can be utilized against output service tax. 2. Tax credit for Education cess of 2% can be utilized for payment of education cess of 2% on output service tax. 3. Tax credit for SHEC of 1% can be utilized for payment of SHEC of 1% on output service tax. Inter-adjustment of Tax credit of Excise Duty, Service Tax is allowed to the service provider because all these taxes are collected by Central Government. Tax credit for Excise Duty, Service Tax is called Cenvat Credit and is regulated through Cenvat Credit Rules, 2004. Question 55: Explain provisions of Rule 6 of Cenvat Credit Rules, 2004. Answer: As per basic principle of cenvat, credit of duty or tax can be availed only for payment of duty on final product or output services. In other words, if no duty is payable on final product or output services, credit of duty/tax paid on inputs or input services cannot be availed. As per Rule 6 of Cenvat Credit Rules 2004, Cenvat credit shall not be allowed on such quantity of input used in or in

relation to the manufacture of exempted goods or for provision of exempted services, or input service used in or in relation to manufacture of exempted goods or for provision of exempted services. If a manufacturer is manufacturing both exempt and dutiable goods (or service provider providing taxable as well as exempt services), it may happen that same inputs/input services are used partly for manufacture of dutiable goods/ taxable services and partly for exempted goods/services. In such cases, the manufacturer/service provider has following 2 options for availing Cenvat credit : (i) Maintain separate Accounts: The manufacturer or service provider of output service shall maintain separate accounts for:(a) receipt, consumption and inventory of input and input service meant for use in the manufacture of dutiable final products or in providing output service and (b) the quantity of input meant for use in the manufacture of exempted goods or service, and take Cenvat credit only on that quantity of input or input service which is intended for use in the manufacture of dutiable goods or in providing taxable output service. (ii) Option not to maintain separate accounts: The manufacturer of goods or the provider of output service, opting not to maintain separate accounts, shall have to pay amount equal to 5% of value of exempted goods (if manufacturer) and of value of exempted services (if service provider). Example 1: Mr. Sachin, a service provider, has provided services of `50,00,000. Out of this, ` 35,00,000 are taxable output services and `15,00,000 are exempt output services. Mr. Sachin has opted not to maintain separate inventory and accounts and pay prescribed amount on value of exempt output services. Service tax paid on his input services, is `3,00,000 plus EC plus SHEC. Rate of service tax, excluding EC and SHEC, is 10%. Calculate the total amount payable including service tax, EC and SHEC by Mr. Sachin by GAR-7 challan. Solution: Calculation of service tax and total amount payable under rule 6 of the CENVAT Credit Rules, 2004 Particulars Amount payable on taxable services Amount payable on exempt services under rule 6 (15,00,000 x 5%) Service tax 3,50,000 75,000 Education cess 7,000 Secondary and higher education cess 3,500 3,500 3,000 500

4,25,000 7,000 Less: CENVAT Credit 3,00,000 6,000 Net tax 1,25,000 1,000 Amount payable by GAR-7 challan = ` 1,25,000 + ` 1,000 + ` 500 = `1,26,500

Notes: (1) Education cess & secondary and higher education cess are not payable on amount payable @ 5% of the exempt services under rule 6 of the CENVAT Credit Rules, 2004.

(2) Credit of education cess and secondary and higher education cess on input services can be utilized only for the payment of education cess and secondary and higher education cess payable on output services (or on excisable goods) respectively. Example 2: Mr. X is engaged in manufacturing of Excisable as well as Exempted goods. During the year following sales were made i) ` 50,00,000 ( Dutiable @ 10.3%) ii) ` 50,00,000 ( Exempted goods) He purchased raw material on which he paid Excise duty of ` 2,50,000 plus education cess and SHEC and receive Services in connection to his business and paid service tax of ` 1,00,000 plus education cess and SHEC. Note: i) Raw material purchased were used to manufacture excisable as well as exempted goods. Compute Cenvat credit available and net duty payable. Solution: Since, manufacturer is engaged in manufacturing excisable as well as exempted goods by using common raw material, in such case, he has two options for availing Cenvat credit as per Rule 6 of Cenvat credit Rule, 2004. iii) Maintain Separate Accounts iv) Pay an amount equal to 5% of value of exempted goods i.e. Particulars Amount payable on taxable goods Amount payable on exempt goods i.e. 5% of `50,00,000 Less: CENVAT Credit (2,50,000 + 1,00,000) Net tax Excise Duty 5,00,000 2,50,000 7,50,000 3,50,000 4,00,000 Education cess 10,000 10,000 7,000 3,000 Secondary and higher education cess 5,000 5,000 3,500 1,500

Question 56: Whether any section of Central Excise Act, 1944 is applicable to Service tax. (For Self Reading Only) Answer: Application of certain provisions of Central Excise Act 1944 Section 83 The provisions of the following section of the Central Excise Act, 1944, as in force from time to time, shall apply, so far as may be, in relation to service tax as they apply in relation to a duty of excise: 9C, 9D, 11B, 11BB, 11C,12 12A, 12B. 12C, 12D, 12E, 14, 14AA, 15, 33A, 35F, 35FF to 35-O (both inclusive), 35Q, 36, 36A, 36B, 37A, 37B, 37C, 37D, 38A and 40. Question 57: Write a note on Appeals to the Commissioner of Central Excise (Appeals). (For Self Reading Only) Answer: Appeals to the Commissioner of Central Excise (Appeals) Section 85 An appeal can be filed to the Commissioner of Central Excise (Appeals) against the orders of Central Excise Officer within

3 months of receiving the orders and appeal should be filed in Form No. ST-4. Commissioner of Central Excise (Appeals) may admit the appeal even after expiry of the period of 3 months provided there were sufficient reason for the delay. Question 58: Write a note on Appeals to Appellate Tribunal (CESTAT). (For Self Reading Only) Answer: Appeals to Appellate Tribunal (CESTAT) Section 86 Appeal can be filed to the Custom Excise Service Tax Appellate Tribunal against the orders of Commissioner of Central Excsie or the order of Commissioner of Central Excise (Appeals). Appeal can be filed maximum within 3 months of the date of receiving the orders however Appellate Tribunal may admit the appeal even after the expiry of the prescribed period provided there were sufficient reasons. Appeal should be filed in Form No. ST-5. Question 59 (V. Imp.): Write a note on valuation of taxable services for charging Service Tax. Answer: Valuation of taxable services for charging service tax Section 67 Value shall be determined in the manner given below: (a) If consideration is in terms of money, the money value of the services e.g. If a TV Programme Production services charges `5,00,000 (exclusive of service tax) as fee from its client, the value of the taxable service rendered will be `5,00,000 and service tax payable shall be `51,500 (5,00,000 x 10.3%). Where the gross amount charged by a service provider, for the service provided or to be provided is inclusive of service tax payable, the value of such taxable service shall be such amount as, with the addition of tax payable, is equal to the gross amount charged. E.g. If total amount charged is `5,00,000 and service tax has not been charged separately, in that case amount of service tax shall be 5,00,000 / 110.3 x 10.3 = `46,690.84 Rounded Off `46,691 (b) If consideration is in kind, market value of such consideration and it will be considered to be inclusive of service tax. E.g. One service provider has rendered services and service recipient has given one gold watch with market value `50,000, in this case amount of service tax shall be 50,000 / 110.3 x 10.3 = `4,669.08 rounded off `4,669 (c) If consideration is partly in cash and partly in kind, it will be total of cash plus market value of consideration in kind and it will be considered to be inclusive of service tax. e.g. A Survey and Map Making Agency provides taxable professional services to one of its clients and it charges `1,00,000 in cash and `75,000 in kind, in this case total consideration shall be `1,75,000 but it will be inclusive of service tax and accordingly amount of service tax shall be 10.3/ 110.3 x 1,75,000 = `16,341.79 Rounded off u/s 37D = `16,342 (d) If any person has rendered free services, no service tax is payable. Question 60: Explain the concept of Pure Agent. Answer: As per Service Tax (Determination of Value) Rules, 2006,

Pure agent means a person who (a) enters into a contractual agreement with the recipient of service to act as his pure agent to incur expenditure or costs in the course of providing taxable service; (b) do not hold any title to the goods or services so procured as pure agent of the recipient of service; (c) does not use such goods or services so procured; and (d) receives only the actual amount incurred to procure such goods or services. The expenditure or costs incurred by the service provider as a pure agent of the recipient of service, shall be excluded from the value of the taxable service. If the service provider is not a pure agent, the value of the taxable service is the total amount of consideration consisting of all components of the taxable service and it is immaterial that the details of individual components of the total consideration is indicated separately in the invoice. Illustration. X contracts with Y, a real estate agent to sell his house and thereupon Y gives an advertisement in television. Y billed X including charges for television advertisement and service tax paid on advertisement in the total bill. In such a case, consideration for the service provided is what X pays to Y. Y does not act as an agent on behalf of X when obtaining the television advertisement even if the cost of television advertisement is mentioned separately in the invoice issued by Y. Advertising service is an input service for the estate agent in order to enable or facilitate him to perform his services as an estate agent Illustration. In the course of providing a taxable service, a service provider incurs costs such as travelling expenses, postage, telephone, etc., and may indicate these items separately on the invoice issued to the recipient of service. In such a case, the service provider is not acting as an agent of the recipient of service but procures such inputs or input service on his own account for providing the taxable service. Such expenses do not become reimbursable expenditure merely because they are indicated separately in the invoice issued by the service provider to the recipient of service and thus shall form part of value of taxable service. Illustration. A contracts with B, an architect for building a house. During the course of providing the taxable service, B incurs expenses such as telephone charges, air travel tickets, hotel accommodation, etc., to enable him to effectively perform the provision of services to A. In such a case, in whatever form B recovers such expenditure from A, whether as a separately itemised expense or as part of an inclusive overall fee, service tax is payable on the total amount charged by B. Value of the taxable service for charging service tax is what A pays to B. Illustration. Company X provides a taxable service of rent-a-cab by providing chauffeur-driven cars for overseas visitors. The chauffeur is given a lump sum amount to cover his food and overnight accommodation and any other incidental expenses such as parking fees by the Company X during the tour. At the end of the tour, the chauffeur returns the balance of the amount with a statement of his expenses and the relevant bills. Company X charges these amounts from the recipients of service. The cost incurred by the chauffeur and billed to the recipient of service constitutes part of gross amount charged for the provision of services by the company X.

Question 61 (V. Imp.): Write a note on Export of Services. Answer: Section 93A/ Export of services, Rules 2005 If any person has exported services from India, no service tax is payable however the payment should be received in India in foreign currency within the time allowed by Reserve Bank of India otherwise exemption is not allowed. If any person has taken input services for the purpose of export of services, service tax paid on such input services shall also be refunded. It will be considered to be export of services if services are provided from India and are used outside India. Similarly if any person has exported goods from India and has paid service tax on the services taken for the purpose of export of goods, service tax paid on such services shall also be refunded. Question 62 (V. Imp.): Explain meaning of Input Service / Output Service. Answer: Input service Rule 2(l) of Cenvat Credit Rules, 2004 means any service, (i)used by a provider of taxable service for providing an output service; or (ii)used by a manufacturer, whether directly or indirectly, in or in relation to the manufacture of final products and clearance of final products upto the place of removal, and includes services used in relation to modernisation, renovation or repairs of a factory, premises of provider of output service or an office relating to such factory or premises, advertisement or sales promotion, market research, storage upto the place of removal, procurement of inputs, accounting, auditing, financing, recruitment and quality control, coaching and training, computer networking, credit rating, share registry, security, business exhibition, legal services, inward transportation of inputs or capital goods and outward transportation upto the place of removal. Output service Rule 2(p) of Cenvat credit Rules, 2004 means any taxable service, provided by the provider of taxable service, to a customer, client, subscriber, policy holder or any other person, as the case may be, and the expressions provider and provided shall be construed accordingly. Question 63 (Imp.): Write a note on Input Service Distributor. Answer: Input service distributor Rule 2(m) of Cenvat Credit Rules, 2004 means an office of the manufacturer or provider of output service, which receives invoices towards purchases of input services and issues invoice, for the purposes of distributing the credit of service tax paid on the said services to its manufacturing units or units providing output service. The ISD must be registered with Service Tax Department.

Credit of service tax attributable to service used in a unit exclusively engaged in manufacture of exempted goods or providing of exempted services shall not be distributed. The credit so distributed should not exceed the amount of service tax paid on input services. Every input service distributor distributing credit shall, issue an invoice, signed by such person for each of the recipient of the credit distributed, and such invoice, shall be serially numbered and shall contain the following namely:(i) the name, address and registration number of the person providing input services and the serial number and date of invoice; (ii) the name and address of the input service distributor; (iii)the name and address of the recipient of the credit distributed; (iv) the amount of the credit distributed.

Question 64 (Imp.): Write a note on liability of service tax in case of supply of goods as well as services. Answer: As per notification No.12/2003 dated 20.06.2003, if any service provider is providing services as well as supplying the goods, in that case, so much of the value of all the taxable services, as is equal to the value of goods and materials sold by the service provider to the recipient of service shall not be chargeable to service tax provided there is documentary proof specifically indicating the value of the said goods and materials. If the goods have been consumed in the process of rendering services, service tax shall be payable on the value of such goods. However, Cenvat credit of duty paid on such goods and materials sold shall not be allowed under the provisions of the CENVAT Credit Rules, 2004 but cenvat credit for the goods consumed shall be allowed. Cenvat credit for capital goods used in providing services and cenvat credit of input services shall be allowed. Example If authorised service station of Maruti Udyog Limited has raised an invoice of `2,00,000 for repair of an accidental car with the bifurcation that `70,000 on account of service charge (labour charge etc) and `1,30,000 is the cost of spare parts and components etc., in this case service tax is payable only on `70,000 but if there is no such bifurcation, service tax is payable on the entire amount of `2,00,000. If there is a bifurcation, cenvat credit of excise duty shall not be allowed in connection with the spare parts etc. of `1,30,000. However, if the assessee has taken any input services, cenvat credit shall be allowed. Similarly, if a coaching institute is charging ` 12,000 as fees including the cost of assignments, in this case service tax is payable on the entire amount of `12,000 but if the coaching institute is providing study material in the form of standard books and price is also printed on such books and a separate invoice has been issued or amount has been shown separately in the same invoice, service tax shall not be charged on the value of study material and also cenvat credit shall not be allowed with regard to such study material i.e. excise duty paid on the value of the paper and other material. If any material has been consumed in the process of rendering services, in that case service tax has to be charged on the

value of such material e.g. One beauty parlour has charged `1,00,000 for bridal makeup and material consumed in the process of rendering service is valued `60,000, in this case service tax should be charged on the entire amount and cenvat credit shall be allowed even for the material consumed in the process. Similarly, if a photography studio has rendered services and has charged `50,000 and has supplied the photographs and cost of photography paper and chemicals etc. is `20,000, in this case service tax shall be charged on the entire amount. Similarly, if a service provider is rendering Photostat services and has charged `1,00,000 which includes cost of paper `70,000, service tax shall be charged on the entire amount. Question 65 (Imp.): Explain concept of Partial Abatement. Answer: Partial Abatement (concession/reduction) Notification No. 1/2006 dated: 01.03.2006 If any service provider is rendering services as well as supplying material, in such cases, service tax shall not be charged on the value of goods or materials supplied as per Notification No.12/2003 but sometimes it is not possible to bifurcate (divide) the value of services and the value of goods, in such cases, service tax shall not be charged on the full amount rather service tax shall be charged only on some part of the total amount i.e. some abatement/concession shall be allowed and only on the balance amount, service tax shall be charged and is as given below. In such cases, Cenvat credit on inputs, input services and capital goods used for providing such taxable services is not allowed. For the purpose of general exemption of `10,00,000, amount before permitting abatement shall be taken into consideration. Abatement has been allowed in case of the following services and balance percentage of the total value taxable is as given below: S. Particulars Abatement Taxable No. % % 1. Mandap Keepers Services (including catering services) 40% 60% 2. Tour Operators Service (a) In relation to booking of accommodation only 90% 10% (b) In respect of tour other than Package Tour (i.e. it 60% 40% will include expenses for stay) (c) In respect of package Tour (i.e. including 75% 25% accommodation, food etc) 3. Convention Services (i.e. services rendered by 40% 60% auditorium or hotel etc. in connection with official meeting) 4. Outdoor catering Services 50% 50% 5. Pandal or Shamiana Contractors Services 30% 70% 6. Services provided by a Restaurant 70% 30% 7. Short-Term Accommodation Services (Services 50% 50% rendered by hotel, inn, guest house etc) 8. Erection Commissioning and installation of Plant and 67% 33% Machinery

9.

Rent-a-cab

60%

40%

Question 66: Write a note on services rendered by Sub-Contractor. Answer: As per Circular No. 96/7/2007 dated 23.08.2007, A sub-contractor is essentially a taxable service provider. The fact that services provided by such sub-contractors are used by the main service provider for completion of his work does not in any way alter the fact of provision of taxable service by the sub-contractor. Services provided by sub-contractors are in the nature of input services. Service tax is, therefore, leviable on any taxable services provided, whether or not the services are provided by a person in his capacity as a sub-contractor and whether or not such services are used as input services. The fact that a given taxable service is intended for use as an input service by another service provider does not alter the taxability of the service provided. Question 67: Explain Recovery of Service Tax. (Self reading) Answer: Recovery of service tax not levied or not paid or erroneously (wrongly) refunded Section 73 If any service tax has not been levied or paid or erroneously refunded, the Central Excise Officer may, give a notice within one year from the relevant date, directing the service provider to pay such service tax however, notice can be given upto 5 years instead of one year if service tax has not been paid because of fraud, collusion, wilful mis-statement, suppression of facts, contravention of any of the provisions with an intention to evade payment of service tax. No notice can be given after expiry of specified time period. Relevant date means, (a) If service tax return has been filed, the date of filing the return and if no return has been filed, the last date for filing service tax return. (b) in a case where any sum, relating to service tax, has erroneously been refunded, the date of such refund. Example If ABC Ltd. has not paid service tax for the month of May 2011, the relevant date shall be 25 th October 2011 and a notice can be given to the company by the service tax department upto one year from the relevant date. i.e. upto 25 th October 2012 If the service tax was not paid as a result of fraud or suppression of facts etc., in that case notice can be given upto five years i.e. upto 25th October 2016. If service tax has been erroneously refunded, the relevant date shall be the date of such refund. Question 68: Explain provisions of Tax Deduction at Source (TDS) with regard to Service Tax. Answer: If any service recipient has to make the payment to the service provider and tax has to be deducted at source, in that case, while deducting tax at source, service tax shall also be included in some of the cases and it will not be included in some other cases and is as given below: TDS excluding Service Tax 1. If the service recipient is making payment of Rent, tax shall be deducted at source under section 194-I @ 10% and tax shall be deducted excluding service tax.

Example ABC Ltd. has let out one commercial building to XYZ Ltd. and rent charged is `20,00,000 plus service tax, in this case, tax to be deducted at source by XYZ Ltd. shall be 20,00,000 x 10% = `2,00,000 and amount payable to ABC Ltd. shall be Rent 20,00,000 Add: Service Tax @ 10.3% 2,06,000 Total 22,06,000 Less: TDS (20,00,000 x 10%) 2,00,000 Amount Payable 20,06,000 2. If the service recipient is making payment of Commission / Brokerage, tax shall be deducted at source under section 194-H @ 10% and tax shall be deducted excluding service tax. Example Mr. X a property dealer has rendered services for getting one commercial building on rent to XYZ Ltd. and Commission charged is `5,00,000 plus service tax, in this case, tax to be deducted at source by XYZ Ltd. shall be 5,00,000 x 10% = `50,000 and amount payable to ABC Ltd. shall be Commission 5,00,000 Add: Service Tax @ 10.3% 51,500 Total 5,51,500 Less: TDS (5,00,000 x 10%) 50,000 Amount Payable 5,01,500 3. If the service recipient is making payment of Commission for Sale of Lottery Tickets , tax shall be deducted at source under section 194-G @ 10% and tax shall be deducted excluding service tax. Haryana Government has given contract for sale of lottery Tickets to XYZ Ltd. and XYZ Ltd. has charged commission of `20,00,000 plus service tax from Haryana Government, in this case, tax to be deducted at source by Haryana Government shall be 20,00,000 x 10% = `2,00,000 and amount payable to ABC Ltd. shall be Commission 20,00,000 Add: Service Tax @ 10.3% 2,06,000 Total 22,06,000 Less: TDS (20,00,000 x 10%) 2,00,000 Amount Payable 20,06,000 4. If the service recipient is making payment of Commission for Insurance Business, tax shall be deducted at source under section 194-D @ 10% and tax shall be deducted excluding service tax. There is a Reverse Charge and service tax has to be paid by the Insurance Company (Such services are called Insurance Auxiliary Services) Example If LIC has to commission of `2,00,000 to their agent Mr. X in connection with insurance business, amount of TDS shall be `2,00,000 x 10% = `20,000 and service tax of `20,600 shall be paid directly by LIC to the Government under Reverse Charge under Rule 2(1)(d). TDS including Service Tax 1. If the service recipient is making payment for Professional / Technical services, tax shall be deducted at source under section 194-J @ 10% and tax shall be deducted including service tax. Example

Mr. X, a Chartered Accountant has given professional services to XYZ Ltd. and charged `20,00,000 plus service tax, in this case, tax to be deducted at source by XYZ Ltd. shall be 22,06,000 x 10% = `2,20,600 and amount payable to Mr. X shall be Professional Charges 20,00,000 Add: Service Tax @ 10.3% 2,06,000 Total 22,06,000 Less: TDS (22,06,000 x 10%) 2,20,600 Amount Payable 19,85,400 2. If the service recipient is making payment for any Contract e.g. Advertising Contract or Catering Contract etc tax shall be deducted at source under section 194-C @ 2% but if payment is being given to any individual or HUF, tax shall be deducted at source @ 1% and tax shall be deducted including service tax. Example ABC Ltd. has rendered advertising services to XYZ Ltd. and charged `20,00,000 plus service tax, in this case, tax to be deducted at source by XYZ Ltd. shall be 22,06,000 x 2% = `44,120 and amount payable to ABC Ltd. shall be Advertising Charges 20,00,000 Add: Service Tax @ 10.3% 2,06,000 Total 22,06,000 Less: TDS (22,06,000 x 2%) 44,120 Amount Payable 21,61,880

INDIVIDUAL SERVICES
Question 69 (V. Imp.): Explain provisions relating to Consulting Engineers Services. Answer: Consulting Engineers Services Section 65(105)(g) (w.e.f. 7th July 1997, Notification No. 23/1997-ST dated 2nd July 1997) Section 65(105)(g) Consulting Engineers Service means any service provided, to any person, by a consulting engineer in relation to advice, consultancy or technical assistance in any disciplines of engineering including the discipline of computer hardware engineering. Section 65(31) - Consulting engineer means any professionally qualified engineer or any body corporate or any other firm who, renders any advice, consultancy or technical assistance in any disciplines of engineering. Services rendered by a consulting engineer in connection with computer hardware engineering as well as software engineering or only in connection with hardware engineering, shall be taxable as consulting engineers services. If services are rendered only for computer software engineering, it will be taxable as information technology service. The scope of the services of a consultant may include: (i) Feasibility report; (ii)Basic design engineering;

(iii)Detailed design engineering; (iv)Construction supervision and project management; (v)Supervision of commissioning and initial operation; (vi)Post-operation and management; (vii)Trouble shooting and technical services, including establishing systems and procedures for an existing plant.

Consulting Engineer will not include those qualified engineers who act as insurance surveyors and loss assessor for insurance companies and therefore service tax levy on the consulting engineer in any discipline of engineering will not cover the insurance surveying and loss assessment services rendered by a qualified engineer. As per rule 7 of Point of Taxation Rule, 2011, service tax shall be payable on actual receipt basis i.e. rule 3 of POT Rules, 2011 shall not be applicable. As per N. N.. 46/2011, dated 19.09.2011, Central Government, hereby exempts the taxable services provided by a consulting engineer to any person on transfer of technology from so much of the service tax leviable thereon under section 66 of the said Act, as is equivalent to the amount of cess paid on the said transfer of technology under the provisions of section 3 of the Research and Development Cess Act, 1986. (Section 3 of The Research and Development Cess Act, 1986 (1) There shall be levied and collected, for the purposes of this Act, a cess at such rate not exceeding five per cent. on all payments made towards the import of technology, as the Central Government may, from time to time, specify, by notification, in the Official Gazette. (2) The cess shall payable to the Central Government by an industrial concern which imports technology on or before making any payments towards such import and shall be paid by the industrial concern to any specified agency.) Illustration 7: Mr. Ramesh, a Consulting Engineer provides the following particulars in respect of various services rendered by him during the quarter ending December 2011: S. No. (i) (ii) (iii) (iv) (v) Particulars Professional advice to one of his friend Consultancy services in computer hardware engineering Technical assistance in computer software engineering and hardware engineering Advice in relation to metallurgical engineering Professional advice to his friend free of charge ` 5,000 15,000 25,000 10,000

Compute the service tax payable by Mr. Ramesh for the quarter ending December, 2011. Service tax has been charged separately by Mr. Ramesh and is not included in any of the receipts mentioned above. Mr. Ramesh is not entitled to the benefit of small service provider available under Notification No. 6/2005 ST dated 01.03.2005. Presume all the payments have been received upto 31st Dec, 2011.

(b) Will your answer be different if the above services are rendered by AB Ltd., a consulting engineering company? Solution: Computation of service tax payable Particulars Professional advice to one of his friend Consultancy services in computer hardware engineering Technical assistance in computer software engineering and hardware engineering Advice in relation to metallurgical engineering Total Value of taxable services Service tax @10% Add: Education cess @ 2% Add: Secondary and higher education cess @ 1% Service tax payable

` 5,000 15,000 25,000 10,000 55,000 55,000 5,500 110 55 5,665

Notes: 1 Advice to friend is taxable as service rendered to any person is taxable. 2.Consultancy and technical assistance in relation to both computer hardware and software engineering are taxable. 3. Advice in relation to any branch of engineering is taxable. 4. Services rendered free of charge are not liable to service tax. (b) Answer will be the same as Consulting Engineer inter alia means any Body Corporate as well. Illustration 8: Mr. X is a Consulting Engineer who is liable to pay service tax and has submitted information as given below: (i) Received advance `2,00,000 inclusive of service tax from ABC Ltd. on 10th April 2011 and services were rendered in September 2011 and bill was issued in October 2011. (ii) Rendered services to Mr. A in October 2011 and a bill of `5,00,000 was issued inclusive of service tax and payment of `3,00,000 was received in February 2012 and balance is yet to be received. (iii) Rendered services to United Nations in December 2011 and payment of `7,00,000 was received in December 2011. No service tax has been collected. (iv) Rendered services to a unit in SEZ and `3,00,000 was received in March 2012 without service tax. (v) Rendered services to foreign diplomatic mission and `5,50,000 was received in January 2012 without service tax. (vi) Rendered services to family members of diplomatic agents and `1,50,000 was received in October 2011 without service tax. Show the tax treatment for Service Tax. Solution:

(i) April to June 2011 Taxable value of services = 2,00,000 Service tax payable = 2,00,000 / 110.3 x 10.3 = 18,676.34 Rounded off under section 37D = 18,676 Last date for making payment shall be 5th July 2011 and if payment is through internet banking, last date shall be 6 th July 2011. (ii) January to March 2012 Total bills raised = 5,00,000 Less: Amount received = 3,00,000 Amount not received = 2,00,000 Since amount charged is inclusive of service tax hence amount of service tax shall be = 3,00,000 / 110.3 x 10.3 = 28,014.51 Rounded off under section 37D = 28,015 Last date for making payment shall be 31st March 2012. (iii) Services provided to United Nations or an international organization is exempt from service tax. (iv) Services provided to a developer of SEZ or a unit of SEZ is exempt from service tax. (v) Services provided to foreign diplomatic mission are exempt from service tax. (vi) Services provided to family members of diplomatic agents are exempt from service tax.

Question 70 (V. Imp.): Explain provisions relating to Mandap Keepers Service. Answer: Mandap Keepers Service Section 65(105)(m) st (w.e.f. 1 July 1997, Notification No. 19/1997-ST dated 26th June 1997) Section 65(105)(m) Mandap Keeper service means any service provided, to any person, by a mandap keeper in relation to the use of mandap in any manner including the facilities provided in relation to such use and also the services, if any, provided as a caterer. As per section 65(66) Mandap means any immovable property as defined in section 3 of the Transfer of Property Act, 1882 and includes any furniture, fixtures, light fittings and floor coverings, photography or video film facilities etc. therein let out for consideration for organizing any official, social or business function; As per Section 65(67) Mandap keeper means a person who allows temporary occupation of a Mandap for consideration for organizing any official, social or business function. Social function include marriage. 1. As per notification no. 1/2006, if the catering services are also provided, value for the purpose of service tax shall be 60% of the gross amount i.e. abatement shall be allowed for 40%.

e.g. if a Mandap Keeper liable to pay service tax and is providing catering service has charged `30,00,000, amount of service tax to be charged by him shall be 30,00,000 x 60% x 10.3% = `1,85,400 2. 3. 4. 5. Hotels and Restaurant which let out their banquets as well as rooms, gardens etc. for holding/organizing any marriage, party, conference shows are covered in the definition of Mandap keeper and are subject to service tax. Renting out of halls etc. for the purpose of holding a dance, drama or music programme or similar competition is also chargeable to Service Tax, in this category. Programmes of dance, drama and music are social functions. Renting out of premises by Art Gallery for exhibition will be exempt from service tax. Services provided by the religious centers as mandap keeper in their precincts (premises) have been exempted from service tax. E.g. Mr. X has arranged marriage of his son in Birla Temple and has paid `21,000, no service tax shall be charged in this case.

If any Tent has been erected on the open ground for the purpose of any such function etc., it will be covered in Pandal and Shamiana services and not in Mandap Keeper Services. Illustration 9: Maharaja Banquets is engaged in providing service in relation to letting out marriage hall and all facilities relating to marriage like catering service. DJ facilities, flower decoration etc. The hall was booked by Mr. A for the marriage of his son on 3rd July, 2011. Maharaja Banquets issued a bill of `20, 00,000(excluding service tax) on 15th July, 2011 for the marriage function and it includes the charges for all other facilities like catering services, DJ services, flower decoration etc. Maharaja Banquets purchased input for `3,00,000 and paid excise duty @ 10% plus EC plus SHEC and has also taken input services for `2,00,000 plus service tax plus EC plus SHEC. Purchased a laptop of `1, 00,000/- plus excise duty@10% plus EC & SHEC for providing DJ services and paid the amount on the same date. Calculate the service tax payable by Maharaja Banquets. Service tax rate applicable is 10.3%. Cenvat credit on laptop purchased is allowed 50% in the year of purchase and the balance in the subsequent year. (b) Calculate the amount of service tax payable if assessee is availing abatement under N.N. 01/2006 and Advise the assessee which option is better for him. Solution (a): Output service tax [20,00,000 x 10.3%] Less: Input tax Credit On inputs (3,00,000 x 10.3%) On input service [2,00,000 x 10.3%] On Laptop [1,00,000 x 10.3% x 50%] 2,06,000 (30,900) (20,600) (5,150)

Net Service tax payable

1,49,350

Solution (b): Assessee is availing abatement under N.N. 01/2006, hence assessee is not eligible to take cenvat credit of input, input service and capital goods and is liable to pay service tax on 60 percent of the gross amount charged. Total Value Charged 20,00,000 Less: Abatement N.N. 01/2006 (40% of gross amount charged) 8,00,000 Taxable value 12,00,000 Service tax @ 10.3% on 12,00,000 1,23,600 Option (b) is better for the assessee as the service tax payable in option (b) is less than the service tax in option (a) Illustration 10: Determine in the following cases whether service tax is payable or not: (i) Mr. A rendered a service which become taxable from 1st June,2011 and issued invoice on 30th April, 2011 and received the amount on 20th May, 2011. Solution: No Service tax is payable on the above service as the invoice and payment are received before the service becomes taxable. (ii) Mr. A received payment of ` 2,00,000(excluding Service tax ) on 1st July,2011 and invoice is issued within 14 days from the date of receiving the payment and the service rendered has become taxable from 10th July, 2011. Solution: As per Rule 5 of Point of Taxation Rules 2011, No service tax shall be payable if the payment has been received before the service becomes taxable and invoice has been issued within a period of 14 days. In the given case, as payment is received before the service is become taxable and invoice is issued within 14 days , so no service tax shall be payable . Illustration 11: Mr. X is engaged in providing Mandap Keeper Services and also he has one Art Gallery and has given the following information for the quarter July to September 2011: (i) Let out the Garden and hall for the purpose of marriage. Issued a bill of ` 10, 00,000/- including charges for catering services on 12th July, 2011. Half the amount is received on 21st July, 2011. (ii) Let out hall for the purpose of holding dance, drama and issued a bill of `3,00,000 on 15th July, 2011 but the amount is not yet received. (iii) Let out the Art Gallery to the artist for the exhibition of work of art. Issued a bill of `2,00,000 on 21st July, 2011. (iv) Let out hall for the purpose of holding dance for `5,00,000 on 25th September, 2011 and issued the bill on 05 th October, 2011. All the figures are excluding service tax. Calculate the value of taxable service and service tax payable for the quarter July to September, 2011. Solution: Computation of Taxable value of service tax under Mandap keeper service for the quarter July to September, 2011

Particulars (i) Letting out garden including catering charges is taxable but only 60% of gross amount charged is chargeable as per Notification no 01/2006 dated 01.03.2006. [10,00,000 x 60%] (ii) Letting out hall for holding dance, drama is a social function and covered under Mandap Keeper Service. (iii) Exhibition of work of arts is not covered under Mandap Keeper Service as per circular no 42/05/2002-ST dated 29.04.2002 (iv) Letting out hall is taxable in the Month of October (Note ) Total taxable value Service tax @ 10.3%

Amount in ` 6,00,000 3,00,000 Nil Nil 9,00,000 92,700

Note: Letting out hall for holding dance is covered under Mandap Keeper Service but as per Point of Taxation Rules if bill is issued within 14 days of completion of service, point of taxation shall be the date of issue of invoice i.e 05.10.2011 and it will be taxable in the month of October.

Question 71(V. Imp.): Explain provisions relating to Practising Chartered Accountant. Answer: Practising Chartered Accountant Section 65(105)(s) (w.e.f. 16th October 1998, Notification No. 53/1998-ST dated 7th Oct 1998) Section 65(105)(s) Practising Chartered Accountant service means any service provided or to be provided, to any person, by a practising chartered accountant in his professional capacity, in any manner. Section 65(83) - Practising Chartered Accountant means a person who is a member of the Institute of Chartered Accountants of India and is holding a certificate of practice granted under the provisions of the Chartered Accountants Act, 1949 and includes any concern engaged in rendering services in the field of chartered accountancy.

Services rendered to any person, by a practising chartered accountant in his professional capacity, in any manner shall be chargeable to service tax. As per Notification No. 32/2011 dated 01.05.2011, if any Chartered Accountant has represented any person in the Court of Commissioner (Appeals) or before Income Tax Appellate Tribunal or before any Statutory Authority, in that case also, it will be subject to service tax. As per rule 7 of Point of Taxation Rule, 2011, service tax shall be payable on actual receipt basis i.e. rule 3 of POT Rules, 2011 shall not be applicable. MEMBERS WHO ARE DEEMED TO BE IN PRACTICE A member of the Institute shall be deemed to be in practice when individually or in partnership with Chartered Accountants in practice, he, in consideration of remuneration received or to be received(i) engages himself in the practice of accountancy; or (ii) offers to perform or performs service involving the auditing or verification of financial transactions, books, accounts or records, or the preparation, verification or certification of financial accounting and related statements or holds himself out to the public as an accountant; or (iii) renders professional services or assistance in or about matters of principle or detail relating to accounting procedure or the recording, presentation or certification of financial facts or data; or (iv) Management Consultancy and other Services and it will include (a) Financial management planning and financial policy determination. (b) Capital structure planning and advice regarding raising finance. (c) Working capital management. (d) Preparing project reports and feasibility studies. (e) Preparing cash budget, cash flow statements, profitability statements, statements of sources and application of funds etc. (f) Budgeting including capital budgets and revenue budgets. (g) Inventory management, material handling and storage. (h) Market research and demand studies. (i) Price-fixation and other management decision making. (j) Management accounting systems, cost control and value analysis.

(k) Control methods and management information and reporting. (l) Personnel recruitment and selection. (m) Setting up executive incentive plans, wage incentive plans etc. (n) Management and operational audits. (o) Valuation of shares and business and advice regarding amalgamation, merger and acquisition. (p) Business Policy, corporate planning, organisation development, growth and diversification. (q) Organisation structure and behaviour, development of human resources including design and conduct of training programmes, work study, job-description, job evaluation and evaluation of work loads. (r) Systems analysis and design, and computer related services including selection of hardware and development of software in all areas of services which can otherwise be rendered by a Chartered Accountant in practice and also to carry out any other professional services relating to EDP. (s) Acting as advisor or consultant to an issue. (t) Any other service as notified by ICAI. Illustration 12: Mr. X a Chartered Accountant, who is liable to pay service tax has submitted particulars as given below: (i) Rendered services in May 2011 and issued bill for `1,20,000 inclusive of service tax. (Out of which `75,000 was received by a cheque on 10th August 2011 and balance on 3rd March 2012.) (ii) Rendered services in the month of June 2011 in connection with scrutiny assessment and a bill of `75,000 was issued and payment was received on 10.10.2011 and amount is inclusive of service tax. (iii) Rendered free services in July 2011 (market value `20,000). (iv) Represented one client in the court of Commissioner (Appeals) in September 2011 and a bill of `1,00,000 was issued and payment was received on 3rd October 2011 and amount is inclusive of service tax. (v) Rendered services to different clients in the month of January 2012 and bills of `7,00,000 was issued inclusive of service tax but only `5,00,000 was received in the same month in full and final settlement. (vi) Received `70,000 in advance inclusive of service tax in March 2012 for services to be rendered in April 2012. Gross receipt is not exceeding `60,00,000 during the year. In the last year he has paid service tax of `2,00,000.

Compute amount of service tax payable for each quarter and also the last date upto which tax should be paid for the financial year 2011-12. (b) If in the above case there is a delay of 10 days in payment of tax in each quarter. Compute interest payable under section 75 and penalty payable under section 76. (c) If the return for the half year ending September 2011 was filed on 18 th October 2011 return for half year ending March 2012 was filed on 27th April 2012, compute penalty payable for each of the return and also determine the last date upto which revised return can be filed. Solution: As per rule 7 of Point of Taxation Rule, 2011, service tax shall be payable on actual receipt basis i.e. rule 3 of POT Rules, 2011 shall not be applicable. April to June 2011 Nil July to September 2011 Since amount charged is inclusive of service tax hence amount of service tax shall be = 75,000 / 110.3 x 10.3 = 7,003.63 Rounded off under section 37D = 7,004 Last date for making payment shall be 5 th October 2011 and if payment is through internet banking, last date shall be 6 th October 2011 October to December 2011 1,75,000 / 110.3 x 10.3 = 16,341.79 Rounded off under section 37D = 16,342 Last date for making payment shall be 5 th January 2012 and if payment is through internet banking, last date shall be 6 th January 2012 January to March 2012 Since amount charged is inclusive of service tax hence amount of service tax shall be = 5,00,000 + 45,000 + 70,000 = 6,15,000 = 6,15,000 / 110.3 x 10.3 = 57,429.73 Rounded off under section 37D = 57,430 Last date for making payment shall be 31st March 2012. Solution (b): April to June 2011 Nil July to September 2011 Interest Payable under section 75

7,004 x 15% x 10/366 = `28.70 = `29 Penalty payment under section 76 1% of the amount of default for 10 days = 1% x 7,004 x 10/31 = `22.59 Or Penalty calculated @ `100 per day for 10 days = `1,000 Penalty liable to be paid is `1,000 October to December 2011 Interest Payable under section 75 16,342 x 15% x 10/366 = `66.98 = `67 Penalty payment under section 76 1% of the amount of default for 10 days = 1% x 16,342 x 10/31 = `52.72 Or Penalty calculated @ `100 per day for 10 days = `1,000 Penalty liable to be paid is `1,000 January to March 2012 Interest Payable under section 75 57,430 x 15% x 10/365 = `236.01 = `236 Penalty payment under section 76 1% of the amount of default for 10 days = 1% x 57,430 x 10/30 = 191.43 Or Penalty calculated @ `100 per day for 10 days = `1,000 Penalty liable to be paid is `1,000 Solution (c): If the return for the half year ending September 2011 was filed on 18 th October 2011, there is no penalty because return is filed within prescribed time period. Revised return can be submitted within a period of 90 days from the date of submission of the original return. 16 th Jan 2012 If return for the half year ending March 2012 was filed on 27th April2012, the penalty shall be `500. Revised return can be submitted within a period of 90 days from the date of submission of the original return. 26 th July 2012 Illustration 13: Ashish Agarwal is a Chartered Accountant. He acquired the certificate of practice from the ICAI in November, 2011. For the quarter ended on March, 2012, his receipts are as follows: (The amounts given below are exclusive of service tax). He is not availing SSP exemption. Particulars Amount

Certification of documents under Export and Import Policy of Government of India Preparation of the financial statement of ABC Ltd. Representation of the client before Income Tax Appellate Tribunal Receipts for tax consultancy provided in the month of December, 2011

(`) 2,00,000 5,00,000 50,000 10,000

He has purchased certain capital goods like computers, laptops, printers, air-conditioners etc. on 01.01.2012 for ` 10,00,000 and has paid excise duty @ 10% plus EC plus Delhi VAT @ 12.5% Using the above information, calculate the value of taxable services for the quarter ended on March, 2012. Tax credit for capital goods shall be allowed to the extent of 100% in the first year itself. Solution: Calculation of the value of taxable services for the quarter ended on March, 2012:Particulars Amount (`) Certification of documents under Export and Import Policy of Government of India 2,00,000 Preparation of the financial statement of ABC Ltd. 5,00,000 Representation of the client before Income Tax Appellate Tribunal 50,000 Receipts for tax consultancy provided in the month of December, 2011 10,000 Total 7,60,000 Service Tax EC @ 2% SHEC @ 1% 7,60,000 x 10.3% 76,000 1,520 760 Less: Cenvat credit i.e. 10,00,000 x 10.3% 1,00,000 2,000 1,000 Balance Cenvat credit to be credit forward 24,000 480 240 If a Chartered Accountant is providing the following services, it will not be subject to service tax 1. Contributing articles to newspapers/magazines 2. Rendering services as author of a book 3. Delivering guest lecture 4. Editing any magazine etc. 5. Other services which are not considered to be services rendered in professional capacity

Question 72 (V. Imp.): Explain provisions relating to Scientific and Technical Consultancy Service. Answer: Scientific and Technical Consultancy Service Section 65(105)(za) (w.e.f. 16th July, 2001, Notification No. 4/2001-ST dated 9th July 2001) Section 65(105)(za) Scientific And Technical Consultancy service means any service provided or to be provided, to any person, by a Scientist or a Technocrat, or any science or technology institution or organisation, in relation to scientific or technical consultancy. Section 65(92) - Scientific or technical consultancy means any advice, consultancy, or scientific or technical assistance, rendered in any manner, either directly or indirectly, by a scientist or a technocrat, or any science or technology institution or organisation, to any person, in one or more disciplines of science or technology. The following points have been clarified vide Letter No. F. NO. B-II/I/2000-TRU, Dated 9-7-2001 - Annexure I. Point raised for clarification Whether services rendered by doctors, medical colleges, nursing homes, hospitals, diagnostic and pathological labs, etc. would come under the purview of the proposed levy. Clarification In common parlance, these categories of service providers are not known as scientists or technocrats or science or technology institutions or organisations. They will not be covered under service tax. Yes. Service tax is liable to be paid when any scientific or technical consultancy service is rendered whether by public funded institutions or by private agencies.

Whether public funded research institutions like CSIR, Indian Council for Agricultural Research, Defence Research and Development Organisation, Indian Institute of Technology and IISC, Regional Engineering Colleges etc., which are exempt from payment of income-tax are covered under the service tax. Whether testing services will be covered under the Mere testing will not attract service tax. proposed levy? However, in case testing is an integral part of the consultancy, then such activity is part and parcel of the taxable service and no abatement of any kind admissible. Many public funded research institutions receive grants or In the facts of this case, no service is

aids from the Government for conducting research/project rendered to any one. Hence the question of work. Whether such activities would be covered under the payment of service tax does not arise. levy? However, if they render service to anyone on payment basis, service tax will be payable on such services. Whether the service tax will be leviable on consultancy If scientific or technical consultancy is provided to Government departments, public sector provided to a Government department for undertakings? which consultation fees are received, then service tax would be applicable. As per rule 7 of Point of Taxation Rule, 2011, service tax shall be payable on actual receipt basis i.e. rule 3 of POT Rules, 2011 shall not be applicable. Illustration 14: Mr. Mohit Agarwal , a technocrat, is providing scientific and technical consultancy services and his receipts are `20,00,000 through taxable services in the financial year 2010-11. He furnishes the following information relating to the services rendered , bills raised, amount received pertaining to this service, for the financial year ended on 31st March, 2012 as underParticulars Advice to Mr. Murali in regard to technical feasibility of a project for `2,00,000 Suggestion to Mrs. Neelu Bansal for improvement in existing technology for `3,00,000 Technical consultancy provided to Government departments for `6,00,000 Services provided to UNICEF, an international organisation in relation to adopting a new technology for `8,00,000 Service rendered on 22.12.2011 06.02.2012 05.01.2012 05.03.2012 Bill raised on 03.01.2012 06.02.2012 22.11.2011 05.03.2012 Amount received on 05.01.2012 15.12.2011 05.01.2012 22.03.2012

Service tax and education cess have been charged separately. 1. Compute service tax liability of Mr. Mohit Agarwal for the financial year 2011-12 and also determine the date upto which service tax should be paid. 2. If the return for the half year ending on September 2011 was filed on compute penalty payable and also determine the last date upto which revised return can be filed. 27 th October 2011

Solution: Since gross receipts in the earlier year has exceeded `10,00,000 hence service tax is payable. Further, since the service provider is an individual, service tax shall be paid on quarterly basis in the manner given below: As per rule 7 of Point of Taxation Rule, 2011, service tax shall be payable on actual receipt basis i.e. rule 3 of POT Rules, 2011 shall not be applicable. April to June, 2011 Nil

July to September, 2011

Nil

October to December, 2011 Services provided to Neelu Bansal 3,00,000 Service tax @ 10% 30,000 Add: Education cess @ 3% 900 Total service tax liability 30,900 Last date for making payment shall be 5 th January 2012 and if payment is through internet banking, last date shall be 6 th January 2012. January to March 2012 Advice to Mr. Murali Technical consultancy to government departments Services provided to UNICEF Taxable value of services Service tax @ 10% Add: Education cess @ 3% Total service tax liability Last date for making payment shall be 31st March 2012. 2,00,000 6,00,000 Nil 8,00,000 80,000 2,400 82,400

2. If return for the half year ending September 2011 was filed on 27th October 2011, the penalty shall be `500. Revised return can be submitted within a period of 90 days from the date of submission of the original return. 25 th January 2012. Note: (1) As per Notification No.16/2002 ST dated 02.08.2002, the Central Government has exempted all the taxable services specified in section 65 of the Finance Act, 1994 provided by any person to the International organizations like UNICEF, from whole of the service tax leviable thereon under section 66 of the Act.

Question 73 (V. Imp.): Explain provisions relating to Commercial Training or Coaching. Answer: Commercial Training or Coaching Section 65(105)(zzc) (w.e.f. 1st July 2003, Notification No. 7/2003-ST dated 20th June 2003) Section 65(105)(zzc) Commercial Training or Coaching service means any service provided or to be provided, to any person, by a commercial training or coaching centre in relation to commercial training or coaching. Commercial Training or Coaching Centre shall include any centre or institute, by whatever name called, where training or coaching is imparted for consideration, whether or not such centre or institute is registered as a trust or a society or similar other organisation under any law for the time being in force and carrying on its activity with or without profit motive and the expression commercial training or coaching shall be construed accordingly. Section 65(26) Commercial training or coaching means any training or coaching provided by a commercial training or coaching centre. Section 65(27) Commercial training or coaching centre means any institute or establishment providing commercial training or coaching for imparting skill or knowledge or lessons on any subject or field other than the sports, with or without issuance of a certificate and includes coaching or tutorial classes.

Services not liable to be taxed 1. As per N.N. 24/2004 dated 10.09.2004, Services rendered by Recreational Training Institute shall be exempt from service tax. Recreational training institute means a commercial training or coaching centre which provides training or coaching relating to recreational activities such as dance, singing, martial arts or hobbies. 2. Services rendered by Vocational Training Institute shall be exempt from service tax. As per Letter No. 334/1/2010 dated 26.02.2010, Vocational training institute means an Industrial Training Institute or an Industrial Training Centre affiliated to the National Council for Vocational Training, offering courses in designated trades as notified under the Apprentices Act, 1961. Some such trades are as given below: 1. Welder (Gas and Electric) 2. Electrician 3. Wireman 4. Auto Electrician 5. Carpenter 6. Plumber 7. Mason (Building Constructor) 8. Furniture and Cabinet Maker 9. Beautician 10. Hair Dresser 11. Health and slimming Assistant 12. Data Preparation & Computer Software 13. Desk Top Publishing Operator 14. Programming and systems Administration Assistant As per N.N. 33/2011 dated 25.04.2011, no service tax shall be charged in case of pre-school coaching or training. Also no service tax shall be charged in case of any coaching or training leading to grant of a certificate or diploma or degree or any educational qualification which is recognised by any law. As per Letter No. 334/3/2011 dated 28.02.2011, if any institute is providing recognised education as well as unrecognised education, in that case unrecognised education shall be taxable. As per N.N. 23/2010 dated 29.04.2010, if any vocational training provider registered under skill development initiative scheme with Director General of Employment and Training and is offering coaching in relation to Modular Employable Skill Courses approved by National Council of Vocational Training, it will be exempt from service tax. Individual imparting home coaching in their independent capacity i.e. not on behalf of a Commercial Training or Coaching Centre. In-house training provided to the employees by their employer in his independent capacity. However, if the employer avails the services of a Commercial Training or Coaching Centre for the purpose of imparting training to his employees, then employer (in his capacity as Service Recipient) will be liable to pay Service Tax to the concerned Commercial Training or Coaching Centre.

3.

4. 5.

6. 6.

Question: ABC technical training institute is providing training for the course like welding, electrician, carpenter etc. it is not recognized by National Council for Vocational Training and has charged `11,00,000. Discuss tax consequences. Answer: The company is liable to pay service tax because it is not recognized by National Council for Vocational Training. If it is a recognized by National Council for Vocational Training, it will be exempt from service tax.

Question 74 (V. Imp.): Explain provisions relating to Technical Testing and Analysis Service. Answer: Technical Testing and Analysis Service Section 65(105)(zzh) st (w.e.f. 1 July, 2003, Notification No. 7/2003-ST dated 20th June 2003) Section 65(105)(zzh) Technical Testing and Analysis service means any service provided or to be provided, to any person, by a Technical Testing and Analysis agency, in relation to technical testing and analysis. Section 65(106) - Technical testing and analysis means any service in relation to physical, chemical, biological or any other scientific testing or analysis of goods or material or information technology software or any immovable property, but does not include any testing or analysis service provided in relation to human beings or animals. Technical Testing and Analysis includes testing and analysis undertaken for the purpose of clinical testing of drugs and formulations; but does not include testing or analysis for the purpose of determination of the nature of diseased

condition, identification of a disease, prevention of any disease or disorder in human beings or animals. Section 65(107) - Technical testing and analysis agency means any agency or person engaged in providing service in relation to technical testing and analysis. Exemption on services provided by Government owned State or District Level Laboratory As per Notification No.6/2006, services provided by Government owned State or District level laboratory in relation to testing and analysis of water quality shall be exempt from service tax. Exemption to approved clinical Research Organizations As per Notification No. 11/2007, dated: 1-3-2007, services provided by Clinical Research Organization approved to conduct clinical trials by the Drugs Controller General of India, in relation to testing and analysis of newly developed drugs, including vaccines and herbal remedies, on human participants so as to ascertain the safety and efficacy of such drugs on human participants shall be exempt from service tax. As per Circular/Letter D.O.F. NO. 334/1/2008-TRU, dated 29-2-2008, Testing and Analysis of IT software shall be included in technical testing and analysis services Exemption to Central and State Seed Testing Laboratories As per Notification No. 10/2010-ST, dated 27-2-2010, services provided by a Central or State Seed Testing Laboratory and Central or State Seed Certification Agency notified under the Seeds Act, 1966 to any person, in relation to technical testing and analysis shall be exempt from service tax. Illustration 15: Mr. X is running an agency of Technical Testing and Analysis service and submitted the information as (a) Provided services to ABC Ltd in relation to biological, chemical scientific testing services on 01.06.2011 for `12,00,000 (exclusive of Service Tax) and bill is issued on same date but ABC Ltd paid the same upto 01.09.2011. (b) Input services taken `4,00,000 plus Service Tax plus EC on 15.06.2011 for providing output taxable service but the payment was not made till year end. (c) It provides services in regard to Testing and Analysis of Information Technology Software of `6,00,000 (exclusive of service tax) and billed on 10.01.2012 but amount is received on 30.04.2012. (d) It provides service for Testing and Analysis in regard to human beings or animals on 31.01.2012 and charged `14,00,000 for the same and payment was received on 15.02.2012. (e) Services provided to UNICEF on 01.02.2012 and charged `5,50,000 and issued bill on 10.02.2012 and amount received for the same on 26.02.2012 (e) It provide service for the purpose of clinical testing of drugs and formulation and received the advance for the same of `3,00,000 (exclusive of service tax) on 15.02.2012 the bill was issued on 01.04.2012 and service was completed on 15.04.2012. (g) Input service taken `2,00,000 plus Service Tax on 18.02.2012 for providing output taxable service and paid the amount on 18.04.2012. (h) Mr. X has made payment of Service Tax on 31stJuly, 2011 for the quarter April to June 2011 through cheque but the

cheque was cleared on 4thAugust, 2011. (i) Mr. X has filed Service Tax return on 31st December, 2011 for the half year ended on 30th September, 2011. Presume gross receipt is not exceeding `60,00,000 and also assessee is not eligible for SSP exemption of `10,00,000. You are required to compute:(i) Service Tax Payable by Mr. X for the F.Y 2011-2012? (ii) Interest payable u/s 75 and penalty u/s 76? (iii) Whether it is mandatory to file Service Tax Return Electronically by Mr. X? (iv) Compute Penalty to be paid for late filing of Service Tax Return. (v) Upto what period the record of Service Tax should be maintained? Solution: April to June, 2011 Services provided to ABC Ltd Service tax @ 10% Add: Education Cess @ 3% Service Tax Liability Less Input Service Tax Credit (4,00,000 x 10.3%) Service Tax Payable ` 12,00,000 1,20,000 3,600 1,23,600 41,200 82,400

July to September, 2011 Service Tax Payable 41,200 Note: Since the Amount of input service is not paid on time i.e. within 3 month of input service taken hence Cenvat Credit taken earlier should be reversed back i.e. 41,200.( Rule 4(7) of Cenvat Credit Rules, 2004. October to December, 2011 January to March, 2012 Services provided for IT software testing Services for testing and analysis on human or animal Services provided to UNICEF Service provided for Clinical testing and drugs Taxable value of services Service tax @ 10% Add: Education cess @ 3% Service Tax Liability Less: Input Service Tax Credit (2,00,000 x 10.3%) Service Tax Payable (ii) NIL 6,00,000 NIL NIL 3,00,000 9,00,000 90,000 2,700 92,700 20,600 72,100

Interest u/s 75 82,400 x 15% x 26/366 = 878.03 Rounded off u/s 37D = 878 Penalty u/s 76 1% of the amount of default for 26 days 82,400 x 1% x 26/31 = 691.09 Rounded Off u/s 37D = 691 or Penalty calculated @ `100 per day for 26 days `2,600 Hence Penalty would be ` 2,600 (iii) Notification No. 43/2011 Every assessee shall submit the half yearly return electronically. Hence Mr. X is required to file Service Tax Return electronically. (iv) Penalty for late filing of Service Tax Return Rule7C (Service Tax Rules, 1994) Upto 30 days `1,000 and after that `100 per day i.e. for 37 days (from 25th November, 2011 to 31st December, 2011) = `3,700 Hence penalty shall be `4,700. (v) According to Rule 5 (Service Tax Rules, 1994) All records should be preserved for at least 5 years immediately after the financial year to which such records pertain. Question: Max Hospital has rendered services for technical testing and analysis services with particulars as given below: 1. Conducted tests for determination of symptoms of various diseases and charged `6,00,000 2. Given vaccinations for prevention of diseases and charged `3,00,000 3. Conducted tests for determination of disorder in human being (blood test, echo-cardiograph, liver function test, lipid profile, TSH, Bone densitometry, kidney test, Treadmill test (TMT), Ultra-sound etc) and charged `25,00,000 They have not collected any service tax. Discuss tax consequences. Answer: As per section 65(106), all the above services are exempt from service tax. Question: Shri Ram Centre for water research has rendered technical testing and analysis services for testing and analysis of water and charged `5,00,000. Discuss tax consequences. Answer: As per Notification No.6/2006, services provided by Government owned State or District level laboratory in relation to testing and analysis of water quality shall be exempt from service tax. Since exemption is given only to the Government laboratory, in the given case, service tax is payable. Question 75 (V. Imp.): Explain provisions relating to Business Exhibition Service. Answer: Business Exhibition Service Section 65(105)(zzo) th (w.e.f. 10 September 2004, vide Finance (No.2), Act, 2004.) Section 65(105)(zzo) Business Exhibition service means any service provided or to be provided, to an exhibitor, by the

organiser of a business exhibition, in relation to business exhibition. Section 65(19a) - Business exhibition means an exhibition, (a) to market; or (b) to promote; or (c) to advertise; or (d) to showcase, any product or service, intended for the growth in business of the producer or provider of such product or service, as the case may be. As per Circular No. 80/10/2004-S.T., DATED 17-9-2004, business exhibition service is a service rendered to an exhibitor by an organizer of a business exhibition that intends to market, promote, advertise or show-case products or services for growth in business of the producers or providers of such products or services. Thus, organizers of events such as trade fairs, road shows, fashion shows, display show-cases kept in airports, railway stations, hotels etc. would be covered under Business Exhibition Service. A display of consumer goods in shops or shopping centres for customers to select and purchase would normally not attract any service tax, as normally no separate charges are collected by the shop-keepers for displaying such goods. However, in case an amount is collected for merely displaying an item, the same would be chargeable to service tax. While event management service (a currently taxable service) also relates to organizing such events, but in that case, the services are rendered to the organizer by an event manager in relation to planning, promoting, organizing etc. Thus, an organizer of a business exhibition is not covered under Event Management Services, but would be covered under Business Exhibition Services. Similarly, while services rendered in relation to a circular, label, documents, hoardings or any other audio visual representation of a product or service falls under advertisement services, the services relating to actual exhibition or display of the product or services would fall under the category of Business Exhibition Services. Illustration 16: ABC Ltd. is engaged in providing Business Exhibition services and the company has submitted the information as given below for the month of September 2011. 1. Services completed for X Ltd. for arranging one fashion show on 10.09.2011 and invoice issued under Rule 4A on 22.09.2011 and amount is `2,00,000 plus service tax and it was received on 10.10.2011. 2. Services completed for Y Ltd. for arranging road show on 01.10.2011 but bill issued on 30.09.2011 and payment was received in advance on 31.08.2011 and amount is `3,00,000 plus service tax. 3. Services completed for Z Ltd. for arranging trade fair on 10.09.2011 but bill issued on 10.10.2011 and payment was received on 03.10.2011 and amount is `4,00,000 plus service tax. Compute amount of service tax payable for the month of September 2011 and also the last date upto which payment should be made.

Service tax paid by the company in the Financial year 2010-11 was `11,00,000. (b) If the payment was delayed by 10 days. Compute interest under section 75 and penalty under section 76. (presume gross receipt is exceeding `60,00,000) Solution: 1. In this case point of taxation shall be 22.09.2011 and service tax should be paid upto 6th October 2011. 2. In this case, point of taxation shall be 31.08.2011, hence service tax should be paid upto 6th September 2011. 3. In this case, point of taxation shall be 10.09.2011 and service tax shall be paid upto 6th October 2011 Amount of service tax to be paid for September 2011 is as given below: Amount of taxable service 2,00,000 Amount of taxable service 4,00,000 Total amount of taxable services 6,00,000 Service Tax @ 10.3% 61,800 th Last date for payment of service tax shall be 6 October 2011. (b) In case of delay for 10 days, interest under section 75 shall be i.e. 61,800 x 18% x 10/366 = `303.93 Rounded off under section 37D = `304 Penalty under section 76 i.e. 100 x 10 = `1,000 Or 61,800 x 1% x 10/31 = `199.35 Penalty shall be `1,000 Illustration 17: Abhi-Ash Services Ltd. Provides Business Exhibition Services to the exhibitors for organizing trade fairs, road shows, fashion shows etc. and submitted the information as follows:v) Provided service to Bachan Ltd. on 01.10.2011 for ` 8,50,000 and bill was issued on the same date. vi) Input services taken of ` 12,00,000 on 02.11.2011 for providing output taxable services and paid the amount on the same date. vii) It has provided services of ` 12,50,000 to Hero Honda Ltd. For road show on 18.12.2011 and billed the same on 20.12.2011. No payment has been received till the end of the year. viii) Services provided to UNICEF on 05.10.2011 and charged ` 5,00,000 and issued bill on 10.02.2012 amount for the same was received on 26.02.2012. ix) It has provided services for which advance was received of ` 30,000 on 14.11.2011. However, the bill was issued on 01.03.2012 and services were completed on 15.03.2012.

x) Input services taken of ` 10,000 on 10.12.2011 for providing output taxable services, but the payment was not made till the end of the year. xi) Company has made payment of Service Tax on 31.01.2012 for the month December 2011 through cheque although the cheque was cleared on 05.02.2012. xii) Company has filed Service Tax Return for the half year ended 31st March, 2012 on 31st December, 2012. (All the above figures are excluding of Service Tax) You are required to compute:xiii) Service Tax Payable by Abhi-Ash Services Ltd. for the months October2011 to December2011? xiv) Interest payable u/s 75 (presume rate to be 15%) and penalty us/ 76? xv) Whether it is mandatory to file Service Tax Return electronically by Abhi-Ash Services Ltd.? xvi) Compute Penalty to be paid for late filing of Service Tax Return. xvii) Upto what period the record of Service Tax should be maintained? Solution: (i) Computation of Tax Liability of Abhi-Ash Services Ltd. October, 2011 Services provided to Bachan Ltd. Service Tax @10% Add: Education Cess @3% Service Tax Payable November, 2011 Advance Received on 14.11.2011 Service Tax @10% Add: Education Cess @3% Service Tax Payable Less: Input Service Tax Credit (12,00,000 x 10.3%) Balance Cenvat credit carried forward December, 2011 Services provided to Hero Honda Ltd. Service Tax @10% Add: Education Cess @ 3% Service Tax Payable Less: Cenvat Credit brought forward Less: Cenvat Credit (10,000 x 10.3%) Balance Service Tax Payable ` 8,50,000 85,000 2,550 87,550 30,000 3,000 90 3,090 1,23,600 1,20,510 12,50,000 1,25,000 3,750 1,28,750 1,20,510 1,030 7,210

Note: (i) Services provided to UNICEF is exempt from Service Tax. (ii) Since the Amount of input service is not paid on time i.e. within 3 month of input service taken hence Cenvat Credit

taken earlier should be reversed back i.e. `1,030 in the Month of March 2012. (ii) Interest u/s 75 7,210 x 15% x 26/366 = 76.83 = 77 Penalty u/s 76 Higher of the following:xviii) @ 100 per day for 26 days = `2,600 Or xix) 7,210 x 1% x 26/31 = ` 60.47 = 60 Hence the penalty would be ` 2,600 xx) Every assessee shall submit the half yearly return electronically. Hence, Abhi-ash services Ltd. required to file Service Tax return electronically. (iv) Penalty for late filing of Service Tax Return Rule7C (Service Tax Rules, 1994) Upto 15 days ` 500 Next 15 days ` 500 On Balance Days (i.e. for 220 days from 26.05.2012 to 31.12.2012 @100 per day) `22,000 Total Penalty Payable `23,000 However, Maximum penalty that can be imposed is ` 20,000. Hence the penalty would be ` 20,000. xxi) According to Rule 5 of Service Tax Rules, 1994 All records should be preserved for atleast period of 5 years, immediately succeeding the financial year to which such records pertains.

Question 76 (V. Imp.): Explain provisions relating to Information Technology Software Service. Answer: Information Technology Software Service Section 65(105)(zzzze) th (w.e.f. 16 May 2008, Notification No. 18/2008-ST dated 10th May 2008) Section 65(105)(zzzze) - Taxable service means any service provided or to be provided, to any person, by any other person in relation to information technology software including, (i)development of information technology software, (ii) study, analysis, design and programming of information technology software, (iii) adaptation, upgradation, enhancement, implementation and other similar services related to information technology software, (iv) providing advice, consultancy and assistance on matters related to information technology software, including conducting feasibility studies on implementation of a system, specifications for a database design, guidance and assistance during the start-up phase of a new system, specifications to secure a database, advice on proprietary information technology software, (v) providing the right to use information technology software for commercial exploitation including right to reproduce, distribute and sell information technology software and right to use software components for the creation of and inclusion in other information technology software products, (vi) providing the right to use information technology software supplied electronically; Section 65(53) "information" has the meaning assigned to it in clause (v) of sub-section (1) of section 2 of the Information Technology Act, 2000 (21 of 2000). Section 65(53a) - "information technology software means any representation of instructions, data, sound or image, including source code and object code, recorded in a machine readable form, and capable of being manipulated or providing interactivity to a user, by means of a computer or an automatic data processing machine or any other device or

equipment. Section 2(1)(v) of the Information Technology Act, 2000 - information includes data, text, images, sound, voice, codes, computer programmes, software and databases or micro film or computer generated micro fiche. Receipts for providing the right to use the packaged software is exempt under Notification No. 53/2010. Illustration 18: Swastik Services Ltd. is carrying on the business of providing Information Technology software services and has provided the following information: xxii) It has provided software upgradation service to ABC Private Ltd. on 12.03.2012 electronically and has charged ` 23,00,000. However, the bill was issued on 20.03.2012. xxiii) It has conducted feasibility study on implementation of a new IT system for SEZ which is completed on 15.01.2012 and received payment of ` 1,50,000 on the same date. xxiv) It has also provided software development service to Mr. Amit for personal use on 18.01.2012 for `1,80,000 and received the payment on the same date. xxv) Swastik Services Ltd. has taken input services of ` 80,000 for providing taxable output services on 15.03.2012.

xxvi) Company has filed Service Tax return for the Half year ended September 2011 on 15.03.2012. (All the above figures are exclusive of Service Tax) Ignore SSP exemption. You are required to:xxvii) Compute Service Tax payable by the Swastik Services Ltd. for the period from 01.01.2012 to 31.03.2012. xxviii) Compute the amount of Penalty on late filing of service tax return by the company. What is the maximum amount of penalty that can be imposed? Solution: (i) Computation of Service Tax payable by Swastik Services Ltd. January 2012 Value of services provided to Mr. Amit Service Tax @ 10% Add: Education Cess @ 3% Total Service Tax payable February 2012 March 2012 Value of Service provided to ABC Pvt. Ltd. Service Tax @ 10% Add: Education Cess @ 3% ` 1,80,000 18,000 540 18,540 NIL 23,00,000 2,30,000 6,900

Total Service Tax payable Less: CENVAT Credit Availed (80,000 x 10.3%) Net Service Tax payable (ii) Penalty for late filing of Service Tax Return Rule7C (Service Tax Rules, 1994) Upto 15 days Next 15 days On Balance Days (i.e. for 112 days from 25.11.2011 to 15.03.2012 @100 per day) Total Penalty Payable Maximum amount of penalty that can be imposed is ` 20,000

2,36,900 8,240 2,28,660 ` 500 500 11,200 12,200

Question: ABC Ltd. is selling various softwares like Quick Heal Anti Virus, Tally, Windows, Office etc. and is not charging service tax. Discuss tax consequence. Answer: No service tax is payable in case of sale of such readymade softwares which is called packaged software rather it is subject to State VAT as per N.N. 53/2010. Question: INFOSYS Limited has rendered services for study, analysis, design and programming of information technology software, outside India and has charged `600 lakhs. They are also rendered services for providing advice and consultancy for the matters of IT Software in India and has charged `400 lakhs. They have not collected any service tax. They have taken input services for rendering such services and has paid `100 lakh plus service tax. Discuss tax consequences. (Ignore SSP exemption) Answer: Services rendered outside India are exempt but services rendered in India are taxable and `400 lakhs shall be considered to be inclusive of service tax and output service tax shall be 400,00,000 x 10.3 / 110.3 37,35,267.00 Company shall be allowed tax credit of `100,00,000 x 10.3% 10,30,000.00 Net Tax Payable shall be 27,05,267.00

También podría gustarte